MARCH 2021 Current Affairs Best Revision Questions

Q) Consider the following statements. a) ISRO successfully launched Brazil’s optical earth observation satellite, Amazonia-1, and 18 co-passenger satellites b) The satellites were carried onboard the PSLV-C51, the 53rd flight of ISRO’s launch vehicle and the first dedicated mission of its commercial arm, NewSpace Ltd.

Which of the above statements is/are correct?

A. 1 only B. 2 only C. Both 1 and 2 D. Neither 1 nor 2

Ans - C

The Indian Space Research Organisation (ISRO) on Sunday successfully launched Brazil’s optical earth observation satellite, Amazonia-1, and 18 co-passenger satellites — five from India and 13 from the U.S. — from the Satish Dhawan Space Centre at Sriharikota. The satellites were carried onboard the PSLV-C51, the 53rd flight of ISRO’s launch vehicle, and the first dedicated mission of its commercial arm, NewSpace India Ltd. The mission was undertaken under a commercial arrangement with Spaceflight Inc., U.S.

Q) Consider the following statements: a) Indian Air Force will perform at an airshow at the Galle Face in Colombo as part of the 70th-anniversary celebrations of the Sri Lankan Air Force (SLAF) b) This is the first performance for the Suryakiran Aerobatic Team (SKAT) outside India since it was resurrected in 2015 with the Hawk advanced jet trainers.

Which of the above statements is/are correct?

A. 1 only

1 TELEGRAM LINK: https://t.me/opdemy WEBSITE: www.opdemy.com

B. 2 only C. Both 1 and 2 D. Neither 1 nor 2

Anc - C

The Suryakiran Aerobatic Team (SKAT) and the Sarang helicopter display team, along with the light combat aircraft, of the Indian Air Force, will perform at an airshow at the Galle Face in Colombo from March 3 to 5 as part of the 70th-anniversary celebrations of the Sri Lankan Air Force (SLAF). This will be the first performance for the SKAT team outside India since it was resurrected in 2015 with the Hawk advanced jet trainers. Earlier, the SKAT team had toured Sri Lanka during the 50th anniversary of the SLAF in 2001.

Q) With reference to the E-Daakhil portal, consider the following statements: 1. E-filing was launched by National Consumer Dispute Redressal Commission (NCDRC) in 2020. 2. Delhi was the first state to implement it.

Which of the statements given above is/are correct?

A. 1 only B. 2 only C. Both 1 and 2 D. Neither 1 nor 2

Ans - C

E-Dakhil portal for Consumer grievance redressal is now operational in 15 States/UTs. The Department of Consumer Affairs is now proactively following up with the states to launch an e-filing portal at their ends. E-filing was launched by National Consumer Dispute Redressal Commission (NCDRC) on 7th September 2020. Delhi was the first state to implement it on 8th September 2020. The Consumer Protection Act, 2019, which has come into force from 20th July 2020, has provision for e-filing of consumer complaints in the Consumer Commissions and online payment of the fees for filing a complaint.

Q) Indian Economy has recorded a 0.4% growth rate in Q3. Which 3 month period is Q3?

A. December-February B. November-January C. October-December D. September-November

Ans - C

Q1 - April, May, and June Q2 - July, August, and September

2 TELEGRAM LINK: https://t.me/opdemy WEBSITE: www.opdemy.com

Q3 - October, November, and December Q4 - January, February, and March

Q) In the heart of which European city, a new sacred building called the House of One is coming up with the aim of bringing Christians, Jews, and Muslims to a single place of worship?

A. Paris B. Berlin C. Rome D. London

Ans - B

Called the House of One or, colloquially, Churmosquagogue, it incorporates a church, a mosque, and a synagogue. The foundation stone will be laid on May 27 by people of these faiths. A meeting space at the center of the building will be open to people of other faiths and world views as well as the secular urban society. The House of One is coming up on the site of the old St Peter’s Church, which was damaged in World War II and completely demolished by the government of East Germany in 1964.

Q) The US has imposed sanctions on which country for the killing of journalist Khashoggi?

A. North Korea B. Saudi Arabia C. Russia D. Sudan

Ans b)

Q) The National Board for Wildlife and Union Ministry of Environment included which medium-sized wildcat found in parts of and Gujarat, in the list of critically endangered species?

A. Ocelot B. Serval C. Lynxes D. Caracal

Ans d)

Though not under grave threat in its other habitats, the animal is on the verge of extinction in India, some experts believe. The recovery programme for critically endangered species in India now includes 22 wildlife species. Besides India, the caracal is found in several dozen countries across Africa, the Middle East, Central, and South Asia. While it flourishes in parts of Africa, its numbers in Asia are declining. The wildcat has long legs, a short face, long canine teeth, and distinctive ears — long and pointy, with tufts of black hair at their tips. The iconic ears are what give the animal its name — caracal comes from the Turkish karakulak, meaning ‘black ears’. In

3 TELEGRAM LINK: https://t.me/opdemy WEBSITE: www.opdemy.com

India is called it siya gosh, a Persian name that translates as ‘black Ear’. A Sanskrit fable exists about a small wild cat named deer ha-Karn or ‘long-eared.

Q) Which Ministry has inaugurated Saras Aajeevika Mela 2021 at Noida Haat?

A. Ministry of Women and Child Development B. Ministry of Rural Development C. Ministry of Skill Development D. Ministry of Housing and Urban Affairs

Ans - B

The Ministry of Rural Development is working to include more women in Self Help Groups (SHGs). He said these SHGs played a vital role in increasing family income and improving quality of life. The Saras Aajeevika Mela 2021 is being organized from 26th February to 14th March 2021 by the Ministry of Rural Development. More than 300 rural self-help groups and craftsmen from 27 states are participating in the Mela.

Q) Which State had become the first in India to present a paperless budget?

A. Kerala B. C. D. Rajasthan

Ans - C

Uttar Pradesh has become the first state in the country to table a paperless budget. The state's the first paperless budget was presented in the state assembly at 11 am on February 22, 2021, by UP State Finance Minister Suresh Khanna. All members of the State Legislature were provided iPads to view the budget highlights, which were also displayed on two big screens put up in the House. The 2021-2022 state was also made available on an App that could be downloaded from Google Play Store with the name 'Uttar Pradesh Sarkar Ka Budget'.

4 TELEGRAM LINK: https://t.me/opdemy WEBSITE: www.opdemy.com

UPSC CSE PRELIMS 2021

CRASH COURSE DATE SUBJECT Budget and Economic 10/04/2021, 11/04/2021 Survey 12/04/2021, 13/04/2021 Art and Culture 14/0/2021, 15/04/2021, Economics 16/04/2021 17/18/2021,18/04/2021, Modern Indian History 19/04/2021 20/04/2021,21/04/2021, Geography 22/04/2021 23/04/2021,24/04/2021, Environment 25/04/2021 26/04/2021,27/04/2021, 28/04/2021,29/04/2021, Polity 30/04/2021 01/05/2021, 02/05/2021 Science and Tech *Note: Also, there will be Lectures on CSAT as and when time permits* PRICE: 1,999 For Any Queries: 7988797680, 7999136838, 6295118940

5 TELEGRAM LINK: https://t.me/opdemy WEBSITE: www.opdemy.com

Q) With reference to the Financial Action Task Force (FATF), consider the following statements: 1. The FATF is an inter-governmental body set up in 1989 to combat money laundering, terror financing, and other related threats to the international financial system. 2. Currently, it has 154 members.

Which of the statements given above is/are correct?

A. 1 only B. 2 only C. Both 1 and 2 D. Neither 1 nor 2

Ans - A

Global money-laundering watchdog the Financial Action Task Force (FATF) decided to keep Pakistan on its terrorism financing “grey list”. FATF President Marcus Player said Pakistan must demonstrate in taking action against UN-designated terrorists and their associates. It also said once Pakistan completes three unfulfilled tasks, the FATF will verify and take a decision on Islamabad's present status in the June plenary. In its plenary held in October last year, the FATF had kept Pakistan on the grey list citing its failure to fulfill six out of 27 obligations of the global money laundering and terrorist financing watchdog.

Q) Consider the following statements. a) Registration and booking for an appointment for COVID vaccination is done through CoWIN Portal — cowin.gov.in. b) There is no Cowin App for beneficiary registration. The App on Play Store is for administrators only

Which of the above statements is/are correct?

A. 1 only B. 2 only C. Both 1 and 2 D. Neither 1 nor 2

Ans - C

Twenty-five lakh potential beneficiaries had registered on the Co-Win portal on Monday, the first day of the second phase of COVID-19 vaccination covering those over 60 and people with

6 TELEGRAM LINK: https://t.me/opdemy WEBSITE: www.opdemy.com

specific comorbidities over the age of 45, the Union Health Ministry said. However, the first day of the wider roll-out saw several glitches in registration, with beneficiaries complaining about unavailability of slots, forcing the Health Ministry to issue a clarification stating that “Registration and booking for an appointment for COVID vaccination is done through CoWIN Portal — cowin.gov.in. There is no CoWin App for beneficiary registration. The App on Play Store is for administrators only.”

Q) Consider the following statements: a) According to the 2021 President’s Trade Agenda and 2020 Annual Report, the revocation of India’s preferential trading status under the Generalised System of Preferences (GSP) program in June 2019 is criticized. b) The Make in India campaign was launched by Prime Minister Modi in 2015 to incentivize production in India.

Which of the above statements is/are correct?

A. 1 only B. 2 only C. Both 1 and 2 D. Neither 1 nor 2

Anc - A

The U.S. tried to resolve “long-standing market access impediments affecting U.S. exporters” during 2020 with India, says the 2021 President’s Trade Agenda and 2020 Annual Report — an annual report submitted by the U.S. Trade Representative (USTR) to Congress. The Make in India's campaign was launched by Prime Minister Modi in 2014 to incentivize production in India. The report describes the Trump administration’s revocation of India’s preferential trading status under the Generalised System of Preferences (GSP) program in June 2019 and the ensuing discussion to achieve a mini trade deal (“package”) throughout 2020.

Q) Which of the following will be the partner country for the three-day Maritime India Summit 2021, which is being organized by the Ministry of Ports, Shipping and Waterways on a virtual platform?

A. Australia B. Thailand C. Russia D. Denmark

Ans - D

The Maritime India Summit 2021 is being organized by the Ministry of Ports, Shipping and Waterways on a virtual platform www.maritimeindiasummit.in from 2nd March to 4th March 2021. Denmark is the partner country for the three-day summit.

Q) The All India Football Federation has confirmed which State is the venue for the forthcoming Hero Indian Women’s League 2020-21 edition?

7 TELEGRAM LINK: https://t.me/opdemy WEBSITE: www.opdemy.com

A. Rajasthan B. Tamil Nadu C. Odisha D. Chhattisgarh

Ans c)

All India Football Federation confirms Odisha as the venue for the forthcoming Hero Indian Women’s League 2020-21 edition. This is the fifth edition of IWL, the division women’s league in India. The Odisha government is a strategic partner of the AIFF (All India Football Federation) as it provides the necessary infrastructure to various men and women age-group teams.

Q) Government has decided to form the Centre of Excellence in gaming and other related areas in collaboration with which IIT?

A. IIT Bombay B. IIT Delhi C. IIT Roorkee D. IIT Kanpur

Ans - A

The government has decided to form a Centre of Excellence in gaming and other related areas in collaboration with IIT Bombay. Addressing the opening ceremony of virtual exhibition and prize announcement of Khel Khel Mein through video conference yesterday, Information and Broadcasting Minister Prakash Javadekar said IIT will start the courses this year itself.

Q) Who will receive the CERAWeek global energy and environment leadership award during an annual CERAWeek Conference-2021?

A. Rahul Gandhi B. Manmohan Singh C. Uddhav Thackray D. Narendra Modi

Ans - D

Prime Minister Narendra Modi will receive the CERAWeek global energy and environment leadership award during an annual CERAWeek Conference-2021. The award is given to him for his commitment to sustainability in the energy and the environment.

Q) India’s PSLV C-51 rocket has successfully launched Amazonia-1 of Brazil. What kind of Satellite is Amazonia-1?

A. Military Satellite B. Earth Observation Satellite

8 TELEGRAM LINK: https://t.me/opdemy WEBSITE: www.opdemy.com

C. Communication Satellite D. Navigation Satellite

Ans - B

India’s Polar rocket has successfully launched Amazonia-1 of Brazil and 18 other satellites from the spaceport here, in the first mission of the year for space agency ISRO. The Polar Satellite Launch Vehicle PSLV-C51 lifted from the first launch pad of the Satish Dhawan Space Centre (SHAR) and first injected into orbit primary payload Amazonia-1. The 637-kg Amazonia-1 which became the first Brazilian satellite to be launched from India is an optical earth observation satellite of the National Institute for Space Research (INPE).

Q) National Science Day (NSD) is celebrated every year on which date to commemorate the discovery of the ‘Raman Effect by Sir Chandrasekhara Venkata Raman, for which he was awarded the Nobel Prize in 1930?

A. 27th February B. 28th February C. 1st March D. 2nd March

As - B

National Science Day (NSD) is celebrated every year on 28th February to commemorate the discovery of the ‘Raman Effect by Sir Chandrasekhara Venkata Raman, for which he was awarded the Nobel Prize in 1930. The first NSD was celebrated in1987. Basic Objective: To propagate the message of the importance of science and its application among the people. 2021 Theme: ‘Future of STI (Science, Technology, and Innovations): Impacts on Education, Skills, and Work’.

Q) Nirmala Sitharaman has attended the G20 Central Bank Governors’ meeting. This meeting is a follow-up for the upcoming 2021 G20 Summit, the sixteenth meeting of Group of Twenty, scheduled to take place in -

A. Vienna, Austria B. Rome, Italy C. Brussels, Belgium D. Hague, Netherlands

Ans - B

The Finance Minister of India, Nirmala Sitharaman has attended the G20 Central Bank Governors’ meeting. This was the first-ever G20 Central Bank Governors’ meeting that was held under the Italian Presidency. The main aim of the meeting was to discuss policy actions for transformative and equitable recovery along with other issues on the agenda including the global economic outlook, financial sector issues, financial inclusion, and sustainable finance. This meeting is a follow-up for the upcoming 2021 G20 Summit, the sixteenth meeting of Group

9 TELEGRAM LINK: https://t.me/opdemy WEBSITE: www.opdemy.com

of Twenty, scheduled to take place in Rome, Italy, on 30–31 October 2021.

Q) Which of the following is the only industrialized nation where having different surnames for married spouses is illegal?

A. Argentina B. Japan C. Norway D. Israel

Ans - B

A section of Japan’s ruling Liberal Democratic Party (LDP) has signaled its opposition to changing a law that forces married couples to have the same surname. Under Japan’s civil code, married couples are required to share the same surname, thus making the country the only industrialized nation where having different surnames for married spouses are illegal.

Q) Consider the following statements. a) Sri Lanka said it will develop the West Container Terminal (WCT) at the Colombo Port, along with India and Japan. b) The Private investor is set to develop the West Container Terminal

Which of the above statements is/are correct?

A. 1 only B. 2 only C. Both 1 and 2 D. Neither 1 nor 2

Ans - C

Sri Lanka on Tuesday said it will develop the West Container Terminal (WCT) at the Colombo Port, along with India and Japan. The decision comes a month after the Rajapaksa government ejected the two partners from a 2019 tripartite agreement to jointly develop the East Container Terminal (ECT), citing resistance to “foreign involvement”. While the High Commission of India had “approved” Adani Ports, which was to invest in the ECT project earlier, Japan is yet to name an investor, according to officially published Cabinet decisions. Neither India nor Japan has officially commented on the offer, or the said private investment from the countries.

Q) Consider the following statements: a) Maharashtra launched the country’s maiden Engineering Research & Development (ER&D) Policy b) The aim is to raise its contribution to the sector in the country to 45% in the next five years.

Which of the above statements is/are correct?

10 TELEGRAM LINK: https://t.me/opdemy WEBSITE: www.opdemy.com

A. 1 only B. 2 only C. Both 1 and 2 D. Neither 1 nor 2

Anc - B

Karnataka launched the country’s maiden Engineering Research & Development (ER&D) Policy on Tuesday to raise its contribution to the sector in the country to 45% in the next five years. The State government anticipates the policy has the potential to create over 50,000 jobs in the ER&D space in five years. According to industry body Nasscom, ER&D has the potential to become a $100-billion industry in the country in the next five years.

Q) With reference to the Land Ports Authority of India (LPAI), consider the following statements: 1. It is a statutory body. 2. It is under the Union Ministry of Finance.

Which of the statements given above is/are correct?

A. 1 only B. 2 only C. Both 1 and 2 D. Neither 1 nor 2

Ans - A

The Land Ports Authority of India (LPAI) celebrated its 9th Foundation Day today at its Headquarter in New Delhi. It is a statutory body (created through the Land Ports Authority of India Act, 2010). Founded in 2012. Parent ministry: Ministry of Home Affairs. Mandate: It is responsible for creating, upgrading, maintaining, and managing border infrastructure in India. It manages several Integrated Check Posts (ICPs) all across Borders of India.

11 TELEGRAM LINK: https://t.me/opdemy WEBSITE: www.opdemy.com

UPSC CSE PRELIMS 2021

CRASH COURSE DATE SUBJECT Budget and Economic 10/04/2021, 11/04/2021 Survey 12/04/2021, 13/04/2021 Art and Culture 14/0/2021, 15/04/2021, Economics 16/04/2021 17/18/2021,18/04/2021, Modern Indian History 19/04/2021 20/04/2021,21/04/2021, Geography 22/04/2021 23/04/2021,24/04/2021, Environment 25/04/2021 26/04/2021,27/04/2021, 28/04/2021,29/04/2021, Polity 30/04/2021 01/05/2021, 02/05/2021 Science and Tech *Note: Also, there will be Lectures on CSAT as and when time permits* PRICE: 1,999 For Any Queries: 7988797680, 7999136838, 6295118940

12 TELEGRAM LINK: https://t.me/opdemy WEBSITE: www.opdemy.com

Q) The United Nations Committee for Development Policy (CDP) has recommended the graduation of which country from the category of Least Developed Country (LDC) A. India B. Pakistan C. Bangladesh D. Nepal

Ans - C

Bangladesh has fulfilled the eligibility criteria in terms of per capita income, human assets, and economic and environmental vulnerability. This is the second consecutive time since 2018 that the CDP has made the recommendation for Bangladesh for graduation from the LDC category. Myanmar and Lao PDR also met the criteria for the 2nd time.

Q) Indian High Commission in Colombo has described which country as its “Priority One” partner in the defense sphere? A. Bangladesh B. Sri Lanka C. Maldives D. Bhutan

Ans - B

Indian High Commission in Colombo has described the island nation of Sri Lanka as its “Priority One” partner in the defense sphere and reiterated the assurance of fullest cooperation in the field of defense and security. The assertion comes at a time when the Sri Lanka Air Force (SLAF) is gearing up for the celebration of its 70th anniversary on 2 March.

Q) Which country has successfully launched its first satellite to monitor the Arctic’s climate and environment called “Arktika-M”?

A. Norway B. The US C. China D. Russia

Ans - D

The Russian space corporation Roscosmos successfully launched its first satellite to monitor the Arctic’s climate and environment. The satellite called “Arktika-M” was launched on February 28, 2021, onboard Soyuz-2.1b carrier rocket, from the Baikonur Cosmodrome in Kazakhstan. This satellite will help to collect information to solve operational meteorology and hydrology problems and monitor the climate and environment in the Arctic region.

Q) Which Ministry will virtually launch the “Sugamya Bharat App”?

13 TELEGRAM LINK: https://t.me/opdemy WEBSITE: www.opdemy.com

A. Ministry of Women & Child Development B. Ministry of Rural Development C. Ministry of Social Justice & Empowerment D. Ministry of Skill Development

Ans - C

The App has been developed by the Department of Empowerment of Persons with Disabilities (DEPwD) under the Ministry of Social Justice and Empowerment. Sugamya Bharat APP — a Crowdsourcing Mobile Application is a means for sensitizing and enhancing accessibility in the 3 pillars of the Accessible India Campaign i.e. built environment, transportation sector, and ICT ecosystem in India.

Q) An iceberg larger than New York City has broken off Antarctica. The crack took place near the Helley research station. This station belongs to which country?

A. Canada B. The USA C. United Kingdom D. France

Ans - C

Halley Research Station is a research facility in Antarctica[2] on the Brunt Ice Shelf operated by the British Antarctic Survey (BAS). The base was established in 1956 to study the Earth's atmosphere. Measurements from Halley led to the discovery of the ozone hole in 1985. The 12-staff station was emptied a month ago before the ice calving happened.

Q) A major multinational air exercise called the "Desert Flag" of the Persian Gulf region with the US, France, India, South Korea, Bahrain, etc. will be held in which country?

A. India B. Saudi Arabia C. UAE D. Qatar

Ans - C

Six Indian Air Force (IAF) Su-30-MKI fighters will take off for the United Arab Emirates (UAE) on March 3 to participate in the multinational exercise 'Desert Flag' to be held there over three weeks. Ten countries, including the US, France, and Saudi Arabia will take part in the high-profile exercise. Two C-17s of the Indian Air Force will also be joining the Indian team for the exercise, IAF officials said.

Q) With reference to the Press Information Bureau, consider the following statements: 1. It is one of the media units working under the Ministry of Information & Broadcasting, Government of India. 2. It was established in 1919 as a small cell under the Home Ministry under the British

14 TELEGRAM LINK: https://t.me/opdemy WEBSITE: www.opdemy.com

government.

Which of the statements given above is/are correct?

A. 1 only B. 2 only C. Both 1 and 2 D. Neither 1 nor 2

Ans - C

Jaideep Bhatnagar has taken over today as Principal Director General, Press Information Bureau. It is one of the media units working under the Ministry of Information & Broadcasting, Government of India. It is the nodal agency for public communication and media relations for the entire Union Government of India. It is based in the National Media Centre, New Delhi. It was established in 1919 as a small cell under the Home Ministry under the British government. In 2019, the PIB set up a fact-checking unit to check government-related news.

Q) Consider the following statements. a) Covaxin shows 81% efficacy, also works against variants b) Covaxin, India’s first COVID-19 vaccine by Serum Institute of India

Which of the above statements is/are correct?

A. 1 only B. 2 only C. Both 1 and 2 D. Neither 1 nor 2

Ans - A

Covaxin, India’s first COVID-19 vaccine, has demonstrated an interim clinical efficacy of 81% in the phase 3 clinical trial, vaccine maker Bharat Biotech said on Wednesday. The vaccine demonstrated high clinical efficacy and significant immunogenicity against variants, the company’s CMD, Krishna Ella, said.

Q) Consider the following statements: a) The U.S. Department of Commerce is preparing to tax aluminum sheet exporters from 18 countries including India. b) The US International Trade Commission (ITC), an independent body to impose anti-dumping or countervailing duties

Which of the above statements is/are correct?

A. 1 only

15 TELEGRAM LINK: https://t.me/opdemy WEBSITE: www.opdemy.com

B. 2 only C. Both 1 and 2 D. Neither 1 nor 2

Anc - C

The U.S. Department of Commerce is preparing to tax aluminum sheet exporters from 18 countries after determining on Tuesday that they had benefited from subsidies and dumping. The US International Trade Commission (ITC), an independent body, must approve the final decision by April 15 to impose anti-dumping or countervailing duties, a department statement said. President Joe Biden’s administration determined that imports from Germany in particular ($287 million in 2019) benefited from dumping, ranging from 40% to 242%.

Q) With reference to the ‘Maritime India Summit 2021’, consider the following statements: 1. During the summit the Sagar-Manthan: Mercantile Marine Domain Awareness Centre has been launched. 2. Sagar-Manthan is an information system for enhancing maritime safety, search and rescue capabilities, security, and marine environment protection.

Which of the statements given above is/are correct?

1 only 2 only Both 1 and 2 Neither 1 nor 2

Ans - C

Prime Minister Narendra Modi today inaugurated the ‘Maritime India Summit 2021’. He informed that the capacity of major ports has increased from 870 million tonnes in 2014 to 1550 million tonnes now. Mega ports with world-class infrastructure are being developed in Vadhavan, Paradip and Deendayal Port in Kandla. India aims to operationalize 23 waterways by 2030. India has as many as 189 lighthouses across its vast coastline and has drawn up a programme for developing tourism in the land adjacent to 78 lighthouses.

Q) The Nag River Pollution Abatement Project has been approved at a cost of Rs. 2,117.54crores. The river flows through which city?

A. Kanpur B. Nagpur C. Mumbai D.

Ans - B

This was announced by Union Minister Nitin Gadkari after chairing a meeting of World Bank officials, DG NMCG, and Nagpur Municipal commissioner. The project, approved under the National River Conservation Plan, will be implemented by the National River Conservation

16 TELEGRAM LINK: https://t.me/opdemy WEBSITE: www.opdemy.com

Directorate, NRCD. It will reduce the pollution level in terms of untreated sewage, flowing solid waste and other impurities flowing into the Nag river and its tributaries.

Q) Lok Sabha TV & Rajya Sabha TV have been merged. Now the channel is called -

A. Parliament TV B. Loktantra TV C. Sansad TV D. Bharat TV

Ans c)

The Rajya Sabha and Lok Sabha Television channels that broadcast the proceedings of the upper and lower Houses of Parliament, respectively, have been merged into a single entity. The new channel has been named the ‘Sansad Television’. This change is effective from March 01, 2021.

Q) World Wildlife Day is observed globally on -

A. 1st March B. 2nd March C. 3rd March D. 4th March

Ans c)

World Wildlife Day is observed globally on the 3rd of March every year to celebrate the beautiful and varied forms of wild fauna and flora present on earth. World Wildlife Day also raises awareness about the benefits that the conservation of wild fauna and flora provides to people living on earth. The day also reminds us of the need to fight against wildlife crime and human-induced reduction of species causing various wide-ranging economic, environmental, and social impacts. World Wildlife Day will be celebrated in 2021 under the theme “Forests and Livelihoods: Sustaining People and Planet”.

Q) World Hearing Day is observed on -

A. 2nd March B. 3rd March C. 4th March D. 5th March

Ans b)

World Hearing Day is held on 3 March each year by the World Health Organisation (WHO) to raise awareness on how to prevent deafness and hearing loss and promote ear and hearing care across the world. The theme of World Hearing Day 2021 is Hearing care for ALL!: Screen,

17 TELEGRAM LINK: https://t.me/opdemy WEBSITE: www.opdemy.com

Rehabilitate, Communicate. World Hearing Day 2021 will mark the launch of the first-ever World Report on Hearing.

Q) Central Revenues Control Laboratory (CRCL) was recognized as a Regional Customs Laboratory (RCL) of the World Customs Organisation (WCO) for Asia-Pacific Region. The CRCL is located in which Indian city?

A. Bangalore B. New Delhi C. Lucknow D. Mumbai

Ans b)

Central Revenues Control Laboratory (CRCL), New Delhi is under the administrative control of Central Board of Indirect Taxes & Customs. Established in 1939, CRCL is the headquarters of 14 Revenue Laboratories, including 2 laboratories working at Government Opium & Alkaloid Works, Ghazipur & Neemuch. With its recognition as RCL, CRCL joins a select group of Customs Laboratories in the region like those in Japan & Korea.

Q) Which Ministry has released the study material of Bhartiya Jnana Parampara (Indian Knowledge Tradition) courses of National Institute of Open Schooling (NIOS) in Noida?

A. Ministry of Culture B. Ministry of Rural Development C. Ministry of Social Justice D. Ministry of Education

Ans - D

NIOS is already making efforts to spread the Indian knowledge tradition in India and abroad. Vedic studies, Sanskrit grammar, Indian philosophy, Sanskrit literature, and Sanskrit language courses have been prepared for Secondary and Senior Secondary level by NIOS on the basis of "Indian knowledge tradition" and their study materials are available to learners in Sanskrit and Hindi language. They are also being translated into English medium and further all these subjects are also being planned to be prepared in major foreign languages to promote the Indian culture and knowledge tradition abroad. The New Education Policy-2020 also emphasizes the creation of a sense of pride towards Indianness in the learner's inner self as well as the establishment of our ancient knowledge, skills, and values.

Q) Recently inaugurated Udaipur science center, is located in:

A. Gujarat B. Odisha C. Rajasthan D. Tripura

Ans - D

18 TELEGRAM LINK: https://t.me/opdemy WEBSITE: www.opdemy.com

The Udaipur Science Centre, at Udaipur, Tripura was dedicated to the people by the Governor of Tripura. Udaipur Science Centre is the 22nd Science Centre which has been developed by the National Council of Science Museums (NCSM) and handed over to the State Governments under the Ministry of Culture’s Scheme for Promotion of Culture of Science. This Science Centre has been developed at a cost of Rs 6 crore funded jointly by the Ministry of Culture, Govt. of India and Dept. of Science, Technology & Environment, Government of Tripura. With this; NCSM has now set up science centers in all the northeastern states.

Q) Consider the following statements. a) The Indian Institute of Science broke into the top 100 in the natural sciences category of the QS World University Rankings by Subject 2021 b) In the broad engineering and technology category, only three institutions made it to the top 100 — the Bombay, Delhi, and Madras IITs

Which of the above statements is/are correct?

A. 1 only B. 2 only C. Both 1 and 2 D. Neither 1 nor 2

Ans - C

The Indian Institute of Science broke into the top 100 in the natural sciences category of the QS World University Rankings by Subject 2021, which were released on Thursday. With regard to subject rankings, there are two new entrants in the top 100 list: JNU’s anthropology programme and O.P. Jindal Global University’s law programme, which is also the only private institution in the list. In the broad engineering and technology category, only three institutions made it to the top 100 — the Bombay, Delhi, and Madras IITs — in comparison to five institutions last year.

Q) Consider the following statements: a) India wants Chabahar port in Afghanistan to be included in the 13-nation International North-South Transport Corridor that extends from India to Russia b) India also wants Afghanistan and Uzbekistan to be included in the International North South Transport Corridor

Which of the above statements is/are correct?

A. 1 only B. 2 only C. Both 1 and 2 D. Neither 1 nor 2

Anc - B

19 TELEGRAM LINK: https://t.me/opdemy WEBSITE: www.opdemy.com

India wants Chabahar port to be included in the 13-nation International North-South Transport Corridor that extends from India to Russia, and expands INSTC membership by including Afghanistan and Uzbekistan said External Affairs Minister S. Jaishankar and Ports and Shipping Minister Mansukh Mandaviya on Thursday. Pitching for Chabahar in the INSTC which goes via Iran’s biggest port Bandar Abbas, Mr. Jaishankar proposed that the land route via Kabul and Tashkent would form the INSTC’s “Eastern corridor”.

Q) Surya Prakash panel, recently seen in the news, was related to:

A. Merger of the Lok Sabha TV (LSTV) and the Rajya Sabha TV (RSTV) B. Decide the appropriate level of reserves that the RBI should hold C. Review the Methodology for Measurement of Poverty D. None of the above

Ans - A

After nearly two years of work, the merger of the Lok Sabha TV (LSTV) and the Rajya Sabha TV (RSTV) has been finalized and will be replaced by Sansad TV. Retired IAS officer Ravi Capoor was appointed Chief Executive Officer (CEO) of Sansad TV. Mr. Capoor’s mandate is also to work out the nitty-gritty of the merger, including the integration of the assets and manpower. In November 2019, a committee headed by former Prasar Bharati Chairman Surya Prakash was set up. It submitted a report in February 2020.

Q) Recently seen in the news, Sri Lanka will develop the West Container Terminal (WCT) at the Colombo Port, along with India and which of the following countries?

A. China B. U.K C. U.S.A D. Japan

Ans - D

Sri Lanka said it will develop the West Container Terminal (WCT) at the Colombo Port, along with India and Japan. The decision comes a month after the Rajapaksa government ejected the two partners from a 2019 tripartite agreement to jointly develop the East Container Terminal (ECT), citing resistance to “foreign involvement”. While the High Commission of India had “approved” Adani Ports, which was to invest in the ECT project earlier, Japan is yet to name an investor.

Q) The National Safety Day (NSD) is celebrated every year on ___ by the National Safety Council (NSC) of India.

A. March 3rd

20 TELEGRAM LINK: https://t.me/opdemy WEBSITE: www.opdemy.com

B. March 4th C. March 5th D. March 6th

Ans - B

The National Safety Day (NSD) is celebrated every year on 4 March by the National Safety Council (NSC) of India. In 2021, we are observing the 50th National Safety Day. The purpose of the day is to instill a sense of safety awareness among people by providing them with safety, health, and environment-related support services. The National Safety Day Theme 2021 is “Sadak Suraksha (Road Safety)”.

Q) An indigenously developed Spectrograph named as Aries-Devasthal Faint Object Spectrograph & Camera (ADFOSC) has been commissioned in -

A. Bangalore B. Hyderabad C. Lucknow D. Nainital

Ans - D

Indian Scientists at Aryabhatta Research Institute of observational sciences (ARIES), Nainital, have indigenously designed and developed a low-cost optical Spectrograph named as Aries-Devasthal Faint Object Spectrograph & Camera (ADFOSC). It has been commissioned on the 3.6-m Devasthal Optical Telescope (DOT), in the Nainital district of Uttarakhand.

Q) Who has topped the 10th edition of Hurun Global Rich List 2021?

A. Mukesh Ambani B. Elon Musk C. Jeff Bezos D. Bill Gates

Ans - B

The 10th Edition of Hurun Global Rich List 2021 was released, which ranked 3228 billionaires from 2,402 companies and 68 countries. The report showed that despite the COVID-19 pandemic, the world added 8 billionaires every week in 2020 and 421 in a year, taking their total number to a record 3,288.

Q) Sri Lanka has announced that it will develop the West Container Terminal (WCT) at the Colombo Port, along with India and which country?

A. The US B. Russia C. Japan D. France

21 TELEGRAM LINK: https://t.me/opdemy WEBSITE: www.opdemy.com

Ans - C

The decision comes a month after the Rajapaksa government ejected the two partners from a 2019 tripartite agreement to jointly develop the East Container Terminal (ECT), citing resistance to “foreign involvement”. While the High Commission of India had “approved” Adani Ports, which was to invest in the ECT project earlier, Japan is yet to name an investor.

Q) Which country has become the 1st country to enter into a pact with India to procure BrahMoscruise missiles?

A. Philippines B. Vietnam C. Saudi Arabia D. Brazil

Ans - A

India and the Philippines have signed a key agreement to facilitate government-to-government deals on military hardware, including the potential supply of BrahMos cruise missiles. The signing of the “implementing arrangement” for procuring defense materials and equipment was announced by the department of national defense of the Philippines on its Facebook page.

Q) With reference to the Himalayan serow, consider the following statements: 1. It is listed as Near-threatened on IUCN Red list. 2. It is listed in CITES Appendix II.

Which of the statements given above is/are correct?

A. 1 only B. 2 only C. Both 1 and 2 D. Neither 1 nor 2

Ans - A

Himalayan serow was spotted in the Manas Tiger Reserve in Assam. It was spotted close to the border with Bhutan in Manas’s Bansbari-Mathanguri forest. The Himalayan serow is a subspecies of the mainland serow native to the Himalayas. An appearance of a goat with long, donkey-like ears, and a habit of standing with forelegs astraddle, make the Serow and ungainly goat antelope. Its coarse (long hair longer than the Goral) varies from black to red.

Q) Consider the following statements. a) Money laundering is the illegal process of making large amounts of money generated by criminal activity, such as drug trafficking or terrorist funding, appear to have come from a legitimate source.

22 TELEGRAM LINK: https://t.me/opdemy WEBSITE: www.opdemy.com

b) The Financial Action Task Force is an intergovernmental organization headquartered in Paris, France

Which of the above statements is/are correct?

A. 1 only B. 2 only C. Both 1 and 2 D. Neither 1 nor 2

Ans - C

Former Jammu and Kashmir Chief Minister and Peoples Democratic Party (PDP) president Mehbooba was on Friday summoned by the Enforcement Directorate in a money laundering case. The money from the criminal activity is considered dirty, and the process "launders" it to make it look clean. The Financial Action Task Force, also known by its French name, Groupe d'action financière, is an intergovernmental organization founded in 1989 on the initiative of the G7 to develop policies to combat money laundering. In 2001, its mandate was expanded to include terrorism financing.

Q) Consider the following statements with reference to Myanmar: a) Myanmar is a Southeast Asian nation bordering India, Bangladesh, China, Laos and Thailand b) Four North-Eastern States viz. Arunachal Pradesh, Nagaland, Tripura, and Mizoram share an international boundary with Myanmar.

Which of the above statements is/are correct?

A. 1 only B. 2 only C. Both 1 and 2 D. Neither 1 nor 2

Anc - A

Four North-Eastern States viz. Arunachal Pradesh, Nagaland, Manipur, and Mizoram share an international boundary with Myanmar. Villagers in Mizoram’s border districts said more than 100 people have crossed over from Myanmar to escape a military crackdown there. Local authorities have, however, confirmed the arrival of 23 people who fled the Tatmadaw – as the Myanmar military is called – that took control of the country following a coup on February 1. At least 50 people have been killed in clashes with the army since. Myanmar shares a long land border of over 1600 Km with India as well as a maritime boundary in the Bay of Bengal.

Q) Ease of Living Index (EoLI) 2020 was published by:

A. NITI Aayog B. Union Ministry of Housing and Urban Affairs C. Association for Democratic Rights

23 TELEGRAM LINK: https://t.me/opdemy WEBSITE: www.opdemy.com

D. Greenpeace

Ans - A

The Minister of Housing and Urban Affairs announced the release of the final rankings of Ease of Living Index (EoLI) 2020. The EoLI aims to measure the well-being of Indian citizens in 111 cities, across the pillars of Quality of Life, Economic-ability, and Sustainability, with 49 indicators under 13 categories. The EoLI 2020 strengthens its scope by consolidating the framework with the addition of a Citizen Perception Survey in the index, holding a weightage of 30%.

Q) Which country is building the world’s first platypus sanctuary?

A. The US B. China C. Russia D. Australia

Ans - D

Australian conservationists have unveiled plans to build the world’s first refuge for the platypus, to promote breeding and rehabilitation as the duck-billed mammal faces extinction due to climate change. The Taronga Conservation Society Australia and the New South Wales State government said they would build the specialist facility, mostly ponds, and burrows for the semi-aquatic creatures, at a zoo 391 km from Sydney, by 2022.

Q) Which Union Territory topped the medals tally in the second edition of the Khelo India Winter National Games?

A. Chandigarh B. Jammu & Kashmir C. Ladakh D. Puducherry

Ans - B

Union Territory of Jammu and Kashmir topped the medals tally in the second edition of the Khelo India Winter National Games. J&K won 11 , 18 silver, and 5 bronze medals. The five-day mega sports event which began on the 26th of February concluded in the famous ski resort of Gulmarg in north Kashmir’s Baramulla district. The event was e-inaugurated by Prime Minister Narendra Modi.

Q) Who has been appointed as the Chairperson of the International Boxing Association (AIBA) Champions and Veterans Committee?

A. Mary Kom B. Vijendra Singh C. Shiva Thappa

24 TELEGRAM LINK: https://t.me/opdemy WEBSITE: www.opdemy.com

D. Akhil Kumar

Ans - A

Six-time world champion boxer Mary Kom has been appointed as the Chairperson of the International Boxing Association (AIBA) Champions and Veterans Committee. The 37-year-old was voted to the post by AIBA’s board of directors on March 03, 2021. The Committee was formed in December 2020. It consists of the most respected worldwide boxing veterans and champions who have achieved significant results and who are ready to share their experience.

Q) Which city emerged as the top city on the government’s Ease of Living Index 2020 that was released by Housing and Urban Affairs Minister Hardeep Singh Puri?

A. Mumbai B. Bangalore C. Hyderabad D. Lucknow

Ans - B

Bengaluru emerged as the top city on the government’s Ease of Living Index 2020 that was released by Housing and Urban Affairs Minister Hardeep Singh Puri. Pune was second and Ahmedabad was ranked third among 111 cities. A total of 111 cities under the Smart Cities program participated in the assessment exercise that was conducted in 2020.

Q) Which statements regarding the recently released Municipal Performance Index (MPI) 2020 arecorrect? a) Indore has topped this list of Million plus population category followed by Surat and Bhopal. b) New Delhi Municipal Council (NDMC) has topped the list of less than a million categories.

A. Only a B. Only b C. Both D. None

Ans C

The Ministry of Housing & Urban Affairs has released the final rankings of the Municipal Performance Index (MPI) 2020. The ranking has classified municipalities into two categories based on their population which are: million-plus category and the less than million categories. The Municipal Performance Index will help Municipalities in better planning and management, filling the gaps in city administration, and improving the liveability of cities for its citizens.

Q) Which Ministry will commemorate ‘Chabahar Day’ on March 4?

A. Ministry of Culture

25 TELEGRAM LINK: https://t.me/opdemy WEBSITE: www.opdemy.com

B. Ministry of Skill Development C. Ministry of External Affairs D. Ministry of Commerce

Ans - C

The Ministry of External Affairs India will commemorate ‘Chabahar Day’ on March 4 on the sidelines of the Maritime India Summit 2021 being held in New Delhi. The virtual event will see the participation of ministers from Afghanistan, Armenia, Iran, Kazakhstan, Russia and Uzbekistan. The Summit is being held from March 2-4, 2021.

Q) With reference to the CSIR Floriculture Mission, consider the following statements: 1. Recently, CSIR Floriculture Mission has been approved for implementation in all states. 2. Floriculture, or flower farming, is a discipline of horticulture concerned with the cultivation of flowering and ornamental plants for gardens and for floristry, comprising the floral industry.

Which of the statements given above is/are correct?

A. 1 only B. 2 only C. Both 1 and 2 D. Neither 1 nor 2

Ans - B

Recently, CSIR Floriculture Mission has been approved for implementation in 21 States/UTs the wherein available knowledge base in CSIR Institutes will be utilized and leveraged to help Indian farmers and industry reposition themselves to meet the import requirements. Floriculture, or flower farming, is a discipline of horticulture concerned with the cultivation of flowering and ornamental plants for gardens and for floristry, comprising the floral industry. This Mission is being implemented in collaboration with the Indian Council of Agricultural Research (ICAR)-Directorate of Floriculture; KVIC; APEDA, TRIFED; Fragrance and Flavour Development Centre (FFDC), Kannauj, Ministry of MSME and Universities.

Q) Consider the following statements. a) China’s new Five-Year Plan (2021-2025) has given the green light for the first dams to be built on the lower reaches of the Yarlung Zangbo river b) The Brahmaputra is known as the Yarlung Zangbo River in Tibet before it flows into India.

Which of the above statements is/are correct?

A. 1 only B. 2 only C. Both 1 and 2 D. Neither 1 nor 2

26 TELEGRAM LINK: https://t.me/opdemy WEBSITE: www.opdemy.com

Ans - C

A draft of China’s new Five-Year Plan (2021-2025), which is set to be formally approved on Thursday has given the green light for the first dams to be built on the lower reaches of Yarlung Zangbo river, as the Brahmaputra is known in Tibet before it flows into India.

Q) With reference to the Overseas Citizens of India, consider the following statements: 1. OCI citizens are of Indian origin but they are foreign passport holders and are not citizens of India. 2. India does not allow dual citizenship but provides certain benefits under Section 7B(I) of the Citizenship Act, 1955 to the OCIs.

Which of the statements given above is/are correct? A. 1 only B. 2 only C. Both 1 and 2 D. Neither 1 nor 2

Anc - C

The Ministry of Home Affairs (MHA) has reiterated through a gazette notification that OCI cardholders can lay claim to “only NRI (Non-Resident Indian) quota seats” in educational institutions based on all-India entrance tests such as NEET, JEE (Mains), JEE (Advanced) or other such all-India professional tests. The notification also said that OCIs are not entitled to undertake any “missionary, mountaineering, journalism and tabligh activities” without prior permission of the Government of India.

Q) With reference to the electronic voting machines (EVMs) consider the following statements: 1. It was introduced in the Paravur Assembly constituency in Ernakulam, Kerala in 1982 polling, a first in the country. 2. In 1992, Parliament inserted Section 61A in the Representation of the People Act, 1950, and rules validating the use of the EVM and paving way for their use in elections.

Which of the statements given above is/are correct?

A. 1 only B. 2 only C. Both 1 and 2 D. Neither 1 nor 2

Ans - A

The use of electronic voting machines (EVMs) in some booths of the Paravur Assembly constituency in Ernakulam, Kerala is etched in the electoral history as they were introduced in

27 TELEGRAM LINK: https://t.me/opdemy WEBSITE: www.opdemy.com

the constituency in the 1982 polling, a first in the country. The EVMs were introduced in 50 polling stations in the election in which the late A.C. Jose of the Congress and the late N. Sivan Pillai of the Communist Party of India were locked in a tight contest. The Congress candidate was defeated by a razor-thin margin of 123.

Q) Prime Minister Narendra Modi has recently participated in the virtual bilateral summit with Stefan Lofven, the Prime Minister of - A. Norway B. Sweden C. Denmark D. Finland

Ans - B

Prime Minister Narendra Modi has participated in the virtual India-Sweden summit, with the Prime Minister of Sweden, Stefan Lofven. The virtual summit was organized to exchange views on regional and global issues and to discuss the bilateral relations between the two countries. This was the fifth interaction between the two leaders since 2015.

Q) Which of the following has conducted a successful flight test of Solid Fuel Ducted Ramjet (SFDR) technology from the integrated test range (ITR), Chandipur, off Odisha coast?

A. HAL B. ISRO C. DRDO D. BDL

Ans C

The Defence Research and Development Organisation (DRDO) conducted a successful flight test of Solid Fuel Ducted Ramjet (SFDR) technology from the integrated test range (ITR), Chandipur, off Odisha coast. The SFDR technology will help DRDO with the technological advantage to develop long-range air-to-air missiles (AAMs). DRDO began developing SFDR first in 2017 and had conducted successful tests in 2018 and 2019 as well.

Q) National Payments Corporation of India (NPCI) has partnered with whom to launch “RuPay SoftPoS” for Indian merchants?

A. Canara Bank B. State Bank of India C. Indian Overseas Bank D. Axis Bank

Ans - B

National Payments Corporation of India (NPCI) and SBI Payments have partnered to launch “RuPay SoftPoS” for Indian merchants. The RuPay SoftPoS solution will provide a cost-effective acceptance infrastructure to retailers at a nominal cost. This innovative solution has the

28 TELEGRAM LINK: https://t.me/opdemy WEBSITE: www.opdemy.com

capability to transform NFC-enabled smartphones into merchant Point of Sale (PoS) terminals for the retailers.

Q) India has held a 1st NSA level Strategic and Counter-Terrorism Dialogue with the NationalSecurity Advisor of which country?

A. The US B. Syria C. Lebanon D. Nigeria

Ans - D

The National Security Adviser (NSA) of Nigeria visited New Delhi for the First Strategic and Counter-Terrorism Dialogue between India and Nigeria at the level of NSA. Within the framework of the close and strategic partnership between India and Nigeria, the National Security Advisors held in-depth discussions on the threats and challenges faced by democratic societies from terrorism, extremism, and radicalization. The two sides identified specific areas of cooperation to enhance their fight against all forms of terrorism, reaffirming their firm belief that there can be no justification for terrorism in any form or manifestation.

Q) Which nation has blocked the shipment of the COVID-19 vaccine of AstraZeneca to Australia?

A. Italy B. Germany C. France D. Denmark

Ans - A

Italy has blocked a shipment of 250,000 doses of the Oxford-AstraZeneca vaccine that was bound for Australia. This comes amid AstraZeneca's failure to deliver the promised doses to the European Union. This is the first time a European Union state has blocked the export of the COVID-19 vaccine amid a global shortfall. The action risks triggering a global backlash against the European Union.

Q) Which State has signed an agreement with Acelor Mittal to invest Rs. 50,000 crores to set up a steel plant?

A. Telangana B. Maharashtra C. Uttar Pradesh D. Odisha

Ans - D

Steelmaker ArcelorMittal Nippon Steel India (AM/NS India) on Thursday inked an agreement

29 TELEGRAM LINK: https://t.me/opdemy WEBSITE: www.opdemy.com

with the Odisha government for setting up a 12 million tonnes per annum (MTPA) greenfield integrated steel plant in Kendrapara district with an investment of Rs 50,000 crore, officials said. Chief Minister Naveen Patnaik and steel tycoon Lakshmi N Mittal were present during the the signing of the Memorandum of Understanding (MoU) at Lokaseva Bhavan, the state secretariat.

Q) What is ShadowPad, recently seen in the news?

A. Malware B. Cryptocurrency C. Exoplanet D. None of the above

Ans - A

On March 3, Maharashtra Power Minister announced that a State Cyber Cell probe had found 14 Trojan horses in the servers of the Maharashtra State Electricity Transmission Company. This malware had the potential to disrupt power distribution in the State. The announcement came in the wake of a report from Recorded Future, a U.S.-based a cybersecurity firm, stating that a group linked to the Chinese government, which is called ‘Red Echo, had targeted 10 vital nodes in India’s power distribution system and two seaports. Recorded Future found a large number of IP addresses linked to critical Indian systems communicating for months with AXIOMATICASYMPTOTE servers connected to Red Echo.

Q) Consider the following statements. a) Indira Sawhney case of 1992 fixed 50%reservation for the marginalized and the poor in government jobs and educational institutions. b) This 50% cannot be relaxed in any certain exceptional and extraordinary situations for bringing far-flung and remote areas' populations into mainstream.

Which of the above statements is/are correct?

A. 1 only B. 2 only C. Both 1 and 2 D. Neither 1 nor 2

Ans - A

The Indira Sawhney judgment had categorically said “50% shall be the rule, and only in certain exceptional and extraordinary situations for bringing far-flung and remote areas population into the mainstream the said 50% rule can be relaxed”. However, over the years, several States, such as Maharashtra and Tamil Nadu, have crossed the Rubicon and passed laws that allow reservation shooting over 60%.

Q) Consider the following statements:

30 TELEGRAM LINK: https://t.me/opdemy WEBSITE: www.opdemy.com

1. Only half of the government schools and anganwadis have tap water supply, despite a 100-day campaign for 100% coverage launched by the Jal Shakti Ministry in October 2020 2. Ministry of Jal Shakti was formed in 2019 merging of two ministries; Ministry of Water Resources, River Development & Ganga Rejuvenation and Ministry of Drinking Water and Sanitation.

Which of the statements given above is/are correct?

A. 1 only B. 2 only C. Both 1 and 2 D. Neither 1 nor 2

Anc - C

Only half of the government schools and anganwadis have tap water supply, despite a 100-day campaign for 100% coverage being launched by the Jal Shakti Ministry in October 2020, according to information provided to the Parliamentary Standing Committee on Water Resources. Less than 8% of schools in Uttar Pradesh and 11% in West Bengal have it, while it is available in only 2-6% of anganwadis in Assam, Jharkhand, Uttar Pradesh, Chhattisgarh, and Bengal.

Q) With reference to International Women's Day (IWD), consider the following statements: 1. International Women's Day (IWD) is celebrated on March 8 every year. 2. This year’s theme is #ChooseToChallenge.

Which of the statements given above is/are correct?

A. 1 only B. 2 only C. Both 1 and 2 D. Neither 1 nor 2

Ans - C

International Women's Day 2021 is being celebrated on the 8th of March with the theme #ChooseToChallenge. It indicates that a "challenged world is an alert world, and from challenge comes change". International Women's Day (IWD) is celebrated on March 8 every year. History: IWD has occurred for well over a century, with the first IWD gathering in 1911 supported by over a million people in Austria, Denmark, Germany, and Switzerland. Prior to this the Socialist Party of America, United Kingdom's Suffragists, and Suffragettes, and further groups campaigned for women's equality.

Q) Which State Govt has approved the formation of a separate school board for nearly 2,700 schools in the city?

31 TELEGRAM LINK: https://t.me/opdemy WEBSITE: www.opdemy.com

A. Puducherry Government B. Delhi Government C. J&K Govt D. Ladakh Government

Ans - B

Delhi government has approved the formation of a separate school board for nearly 2,700 schools in the city. There are around 1,000 Delhi government schools and about 1,700 private schools, most of them affiliated to the CBSE, in the city. The new Board will have a governing body headed by the Education Minister of the Delhi government and an executive body headed by a chief executive officer.

Q) Which Union Minister has addressed the inaugural session of TechBharat 2021?

A. S. Jaishankar B. Nitin Gadkari C. Prakash Javadekar D. Dr. Harsh Vardhan

And - D

Union Minister for Health & Family Welfare, Dr. Harsh Vardhan has addressed the inaugural session of TechBharat 2021. Laghu Udyog Bharati and IMS Foundation have organized the 2nd edition of the conclave bringing together stakeholders from the HealthTech & Edutech sector on a virtual platform.

Q) The Cyberabad Police has launched India’s first-ever Transgender Community Desk’ at the Gachibowli Police Station in which State?

A. Assam B. Maharashtra C. Telangana D. Chhattisgarh

Ans c)

The Cyberabad Police has launched India’s first-ever Transgender Community Desk’ at the Gachibowli Police Station in Hyderabad, Telangana. This desk is the first-of-its-kind gender-inclusive community policing initiatives in the country. The desk was formally inaugurated on Saturday by Cyberabad Police chief VC Sajjanar in a ceremony that was attended by over 200 transgender people.

Q) What is the theme for International Women's Day 2021?

A. Choose to change

32 TELEGRAM LINK: https://t.me/opdemy WEBSITE: www.opdemy.com

B. Choose to challenge C. Choose to be strong D. Choose to be you

Ans - B

International Women's Day 2021 is being celebrated on the 8th of March with the theme #ChooseToChallenge. It indicates that a "challenged world is an alert world, and from challenge comes change". IWD has occurred for well over a century, with the first IWD gathering in 1911 supported by over a million people in Austria, Denmark, Germany, and Switzerland. Prior to this the Socialist Party of America, United Kingdom's Suffragists and Suffragettes, and further groups campaigned for women's equality.

Q) Digital platform ‘Jagrut Tripura’ launched by CM to boost what?

A. Export B. Infrastructure C. E-Governance D. State News

Ans - C

The Tripura government has come up with a digital platform ‘Jagrut Tripura’ to help people get benefits from various schemes of the Centre and the state government. At least 102 schemes of various departments of both governments are available on the platform. The ‘Jagrut Tripura’ will empower citizens of the northeastern state. The digital platform is one of the many proactive initiatives undertaken by the state government to spur a technology-led innovation ecosystem to make ‘Atmanirbhar’ (self-reliant) Tripura.

Q) Indian Railway has integrated all railway helplines into a single number which is -

A. 139 B. 127 C. 144 D. 152

Ans - A

Indian Railway has integrated all railway helplines into a single number 139 which is Rail Madad Helpline for quick grievance redressal and inquiry during the journey. As the new helpline number 139 will take over all the existing helpline numbers, it will be easy for the passengers to remember this number and connect with Railways for all their needs during the travel.

Q) Singorgarh Fort, a hill-fort of Gondwana Kingdom is located in:

A. Rajasthan B. Uttar Pradesh

33 TELEGRAM LINK: https://t.me/opdemy WEBSITE: www.opdemy.com

C. D. Madhya Pradesh

Ans - D

The President of India Ram Nath Kovind laid the foundation stone for the conservation works of Singorgarh Fort in Madhya Pradesh. The President also inaugurated the newly carved Jabalpur Circle of Archaeological Survey of India. Singorgarh Fort is located in Singrampur village of Damoh district in Madhya Pradesh. It is a hill-fort of Gondwana Kingdom, spread over the hills of a forested area.

Q) With reference to the Unlawful Activities (Prevention) Act (UAPA), Consider the following statements. a) UAPA was passed in 1967 with an aim to effective prevention of unlawful activities associations in India. b) Under UAPA, both Indian and foreign nationals can be charged.

Which of the above statements is/are correct?

A. 1 only B. 2 only C. Both 1 and 2 D. Neither 1 nor 2

Ans - C

There have been over 72% increase in the number of persons arrested under the UAPA (Unlawful Activities [Prevention] Act) in 2019 compared to the year 2015, data provided by the Ministry of Home Affairs (MHA) in the Lok Sabha show. UAPA was passed in 1967. It aims at effective prevention of unlawful activities associations in India Under UAPA, both Indian and foreign nationals can be charged. It will be applicable to the offenders in the same manner, even if the crime is committed on foreign land, outside India.

Q) Consider the following statements: 1. Cybercrime grievance portal- cybercrime.gov.in, aims to raise a group of “cybercrime volunteers” to flag “unlawful content” on the Internet. 2. It has been launched by the Ministry of Home Affairs (MHA)

Which of the statements given above is/are correct?

A. 1 only B. 2 only C. Both 1 and 2 D. Neither 1 nor 2

Anc - C

34 TELEGRAM LINK: https://t.me/opdemy WEBSITE: www.opdemy.com

The Ministry of Home Affairs (MHA) informed the Lok Sabha on Tuesday that a “cyber volunteer” programme has been rolled out for “cyber hygiene promotion” and the services of volunteers would be utilized by the State police as per requirement. The MHA, through its cybercrime grievance portal- cybercrime.gov.in, aims to raise a group of “cybercrime volunteers” to flag “unlawful content” on the Internet.

Q) Where is India’s 1st Forest Healing Centre located, which was inspired by the Japanese technique of forest bathing for stress relief & mood rejuvenation?

A. Sikkim B. Uttarakhand C. Assam D. Himachal Pradesh

Ans - B

India’s 1st Forest Healing centre was inaugurated by noted environmentalist Joginder Bisht in the pine-dominated forest located at Ranikhet in Kalika Uttarakhand. It was developed by the Research Wing of Uttarakhand Forest Department in an area of around 13 acres. The forest is inspired by the Japanese technique of forest bathing (shinrin-yoku) and ancient Indian traditions. It is themed on ‘be silent, go slow, think less, and feel more.

Q) With reference to the Azadi Ka Amrit Mahotsav, consider the following statements: 1. The 5 pillars for the celebration of the 75 years of independence are Freedom Struggle, Ideas at 75, Achievements at 75, Actions at 75, and Resolve at 75. 2. Recently a committee was formed by the central government to commemorate the 75 years of India's independence which is led by Prime Minister Modi

Which of the statements given above is/are correct?

A. 1 only B. 2 only C. Both 1 and 2 D. Neither 1 nor 2

Ans C

Prime Minister Modi led National Committee to commemorate 75 years of independence, Azadi Ka Amrit Mahotsav held its first meeting. The Prime Minister informed that 5 pillars have been decided for the celebration of the 75 years. These are Freedom Struggle, Ideas at 75, Achievements at 75, Actions at 75 and Resolve at 75. Various members of the National Committee attended the meeting including Governors, Union Ministers, Chief Ministers, political leaders, scientists, officials, media personalities, spiritual Leaders and eminent persons from other walks of life.

35 TELEGRAM LINK: https://t.me/opdemy WEBSITE: www.opdemy.com

Q) Across which river was the 200 feet “The Bridge of Compassion”, was built by Border Road Organisation in a record 26 days?

A. Yamuna River B. Ghaghara River C. Alakananda River D. Rishiganga River

And D

The Border Road Organisation (BRO) has built a 200 feet bridge across the Rishiganga river, connecting 13 villages in Uttarakhand. The bridge was built by BRO in a record 26 days, as a replacement to the 90-meter Reinforced Cement Concrete (RCC) bridge that was washed away in the February 7 flash floods in the Rishiganga river. As a tribute to the sacrifice of the Power plant workers, the bridge was named “The Bridge of Compassion”.

Q) What is India’s Rank in the “UNCTAD B2C E-COMMERCE INDEX 2020”, that measures the economic preparedness of the countries to support online shopping?

A. 74th B. 72nd C. 71st D. 73rd

Ans - C

In accordance with the ‘UNCTAD B2C E-COMMERCE INDEX 2020- Spotlight on Latin America and the Caribbean’, India ranked 71 out of 152 countries as compared to 75th position in 2019. The index measures an economy’s preparedness to support online shopping where B2C stands for Business to Consumer. UNCTAD is an acronym for the United Nations Conference on Trade and Development. The index has been topped by Switzerland.

Q) With reference to the World Heritage Sites, consider the following statements: 1. At present, India has 78 World Heritage Properties. 2. ‘Dholavira: A Harappan City’ is the first World Heritage Sites in India.

Which of the statements given above is/are correct?

A. 1 only B. 2 only C. Both 1 and 2 D. Neither 1 nor 2

Ans D

Union Minister of Culture and Tourism informed Lok Sabha about the Declaration Of World Heritage Sites By UNESCO. At present, India has 38 World Heritage Properties. All the sites under the Ministry are

36 TELEGRAM LINK: https://t.me/opdemy WEBSITE: www.opdemy.com

conserved as per ASI’s Conservation Policy. At present, India has 42 sites listed under the Tentative List which is a prerequisite condition for inscription as a World Heritage Site.

Q) Which country is planning (as of Mar’21) to establish a Center of Excellence for the dairy sector in Rajasthan?

A. Sweden B. Norway C. Denmark D. Switzerland

Ans - C

On March 06, 2021, Denmark Ambassador Freddy Svane had a meeting with Rajasthan CM Ashok Gehlot and Governor Kalraj Mishra to set up a Center of Excellence for the dairy sector in Rajasthan. Rajasthan is the second-highest milk-producing state in India contributing around 11% of the total milk production.

Q) Trishul Military Airbase, recently seen in the news, is located in:

A. Uttar Pradesh B. Bihar C. Gujarat D. Punjab

Ans - A

Minister of Civil Aviation flagged off the first flight from Delhi to the newly upgraded Trishul Military Airbase, Bareilly Airport, Uttar Pradesh. The Bareilly airport has been upgraded for commercial flight operations under the Regional Connectivity Scheme – Ude Desh Ka Aam Nagrik (RCS-UDAN) of the Government of India. Trishul Military Airbase, Bareilly belongs to the Indian Air Force and the land was handed over to the Airport Authority of India for construction of the interim civil aviation operations.

37 TELEGRAM LINK: https://t.me/opdemy WEBSITE: www.opdemy.com

UPSC CSE PRELIMS 2021

CRASH COURSE DATE SUBJECT Budget and Economic 10/04/2021, 11/04/2021 Survey 12/04/2021, 13/04/2021 Art and Culture 14/0/2021, 15/04/2021, Economics 16/04/2021 17/18/2021,18/04/2021, Modern Indian History 19/04/2021 20/04/2021,21/04/2021, Geography 22/04/2021 23/04/2021,24/04/2021, Environment 25/04/2021 26/04/2021,27/04/2021, 28/04/2021,29/04/2021, Polity 30/04/2021 01/05/2021, 02/05/2021 Science and Tech *Note: Also, there will be Lectures on CSAT as and when time permits* PRICE: 1,999 For Any Queries: 7988797680, 7999136838, 6295118940

38 TELEGRAM LINK: https://t.me/opdemy WEBSITE: www.opdemy.com

Q) Which application recently (in Mar’21) collaborated with the National Skill Development Corporation (NSDC)’s eSkill India portal to promote nano-entrepreneurship amongst the financial service sector?

A. i-mobile pay B. E-Gyan Mitra C. SahiPay D. DakPay

Ans - D

The National Skill Development Corporation (NSDC) collaborated with SahiPay, an Android-based integrated platform developed by Manipal Business Solutions (MBS) to promote nano-entrepreneurship amongst youth in the financial services sector. As a part of the collaboration, candidates will be provided access to free digital skilling on the ‘Online Entrepreneurship Program’ through NSDC’s eSkill India portal.

Q) Consider the following statements. a) Garhwal MP Tirath Singh Rawat took oath as the Chief Minister of Uttarakhand b) The governor of Uttarakhand is Baby Rani Maurya.

Which of the above statements is/are correct?

A. 1 only B. 2 only C. Both 1 and 2 D. Neither 1 nor 2

Ans - C

Garhwal MP Tirath Singh Rawat on Wednesday took oath as the Chief Minister of Uttarakhand at the Raj Bhavan in Dehradun after the BJP Legislature Party elected him its leader. His predecessor, Trivendra Singh Rawat, resigned on Tuesday.

Q) Consider the following statements: 1. The country’s biggest floating solar power plant to date, by generation capacity is being developed by the NTPC 2. With an installation capacity of 447 MW, it is being built at Ramagundam in Peddapalli district, Telangana

Which of the statements given above is/are correct?

39 TELEGRAM LINK: https://t.me/opdemy WEBSITE: www.opdemy.com

A. 1 only B. 2 only C. Both 1 and 2 D. Neither 1 nor 2

Anc - C

The country’s biggest floating solar power plant to date, by generation capacity, which is being developed by the NTPC in the reservoir of its thermal plant at Ramagundam in Peddapalli district, Telangana, is set to be commissioned by May-June next. Work on the 100-megawatt plant is in the final stages of completion. This will be one of the renewable (solar) energy plants being developed by the NTPC with an installed capacity of 447 MW in the southern region and the entire capacity will be commissioned by March 2023.

Q) The bridge ‘Maitri Setu’ has been built over which of the following river?

A. Ganga B. Teesta C. Damodar D. Feni

Ans - D

Prime Minister Narendra Modi inaugurated ‘Maitri Setu’ between India and Bangladesh The bridge ‘Maitri Setu’ has been built over the Feni river which flows between the Indian boundary in Tripura State and Bangladesh. The 1.9 Km long bridge joins Sabroom (in Tripura) with Ramgarh (in Bangladesh). The name ‘Maitri Setu’ symbolizes growing bilateral relations and friendly ties between India and Bangladesh.

Q) Which State/UT has launched “Super-75” scholarship schemes for meritorious girls, on the occasion of International Women’s Day 2021?

A. New Delhi B.Jammu & Kashmir C. Assam D. Maharashtra

Ans - B

The Jammu and Kashmir Lieutenant Governor Manoj Sinha has launched “Super-75” scholarship schemes for meritorious girls, on the occasion of International Women’s Day 2021. The basic aim of this scholarship scheme is to facilitate the education and entrepreneurship of women.

40 TELEGRAM LINK: https://t.me/opdemy WEBSITE: www.opdemy.com

Q) The Heritage Foundation, a US Conservative think-tank, recently launched the “Economic Freedom Index”. Which of the following statements about the index is/are correct? i) Singapore topped the list. ii) India ranked 126th.

A. Only a B. Only b C. Both D. None

Ans - C

The Heritage Foundation, a US Conservative think-tank, recently launched the “Economic Freedom Index”. The index was prepared by covering 184 countries this time for the period of July 2019 to June 2020. This is the first time the index has been published since the COVID-19 pandemic overturned lives as well as global economies and the scoring reflects to a small extent how governments have responded to the health crisis.

Q) Prime Minister Shri Narendra Modi has inaugurated the ‘Maitri Setu’ bridge between India and which country?

A. Nepal B. Pakistan C. Myanmar D. Bangladesh

Ans - D

Prime Minister Shri Narendra Modi has inaugurated the ‘Maitri Setu’ bridge between India and Bangladesh through a video conference, to strengthen the connectivity between both countries and especially with the northeastern region of India. Maitri bridge will also make Tripura the ‘Gateway of North East’ with access to Chittagong Port of Bangladesh, which is just 80 km from the Sabroom.

Q) Which European country has recently voted in favor of a proposal banning full facial coverings including the and niqab in almost all public places?

A. France B. Switzerland C. Austria D. Denmark

Ans - B

Switzerland has voted in favor of a proposal banning full facial coverings including the burqa and niqab in almost all public places. In the public referendum, around 51.21 percent of voters voted in support of the controversial proposal. The vote means that full facial covering will be

41 TELEGRAM LINK: https://t.me/opdemy WEBSITE: www.opdemy.com

banned in all publicly accessible places in Switzerland including on the streets, public transport, in public offices such as restaurants, shops, and in the countryside.

Q) Which State/UT presented a Rs 69,000-crore budget themed on ‘patriotism’ or ‘Deshbhakti’ for the financial year 2021-22?

A. New Delhi B. Maharashtra C. Gujarat D. Uttar Pradesh

Ans - A

The Delhi government presented a Rs 69,000-crore budget themed on ‘patriotism’ or ‘Deshbhakti’ for the financial year 2021-22. The budget was presented by the Deputy Chief Minister of Delhi, Manish Sisodia. While presenting the budget, he announced that the the government has decided to celebrate the 75th Independence Day of India. The Delhi the government will hold programmes from March 12, 2021, which will last for 75 weeks.

Q) Which sportsperson has won the BBC Indian Sportswoman-of-the-Year award?

A. Hima Das B. Smriti Mandhana C. Koneru Humpy D. PV Sindhu

Ans - C

World rapid chess champion Koneru Humpy has won the BBC Indian Sportswoman-of-the-Year award. BBC director-general Tim Davie, hosting the virtual awards ceremony. The Lifetime The achievement award went to veteran athlete Anju Bobby George.

Q) With reference to the UAPA (Unlawful Activities [Prevention] Act), consider the following statements: 1. Cases under the UAPA are investigated only by the National Investigation Agency (NIA). 2. Under the UAPA, getting bail is rare and the investigating agency has up to 365 days to file a charge sheet.

Which of the statements given above is/are correct?

A. 1 only B. 2 only C. Both 1 and 2 D. Neither 1 nor 2

Ans - D

There have been an over 72% increase in the number of persons arrested under the UAPA

42 TELEGRAM LINK: https://t.me/opdemy WEBSITE: www.opdemy.com

(Unlawful Activities [Prevention] Act) in 2019 compared to the year 2015, data provided by the Ministry of Home Affairs (MHA) in the Lok Sabha show. As many as 1,948 persons were arrested under the UAPA in 1,226 cases registered across the country in 2019. From 2015 till 2018, the cases registered under the Act annually stood at 897, 922, 901 and 1,182 respectively, while the number of arrests was 1,128, 999, 1,554 and 1,421. In 2019, the highest number of such cases were registered in Manipur (306), followed by Tamil Nadu (270), Jammu & Kashmir (255), Jharkhand (105), and Assam (87) cases.

Q) Consider the following statements. a) The QUAD virtual summit is to take place today where leaders of the Indo-Pacific group are meeting for the first time. b) Access to COVID-19 vaccines, cooperation on technology, and climate change are on the top of the agenda

Which of the above statements is/are correct?

A. 1 only B. 2 only C. Both 1 and 2 D. Neither 1 nor 2

Ans - C

Access to COVID-19 vaccines, cooperation on technology, and climate change are on the top of the agenda as Prime Minister Narendra Modi will join U.S. President Joseph Biden, Australian PM Scott Morrison and Japanese PM Yoshihide Suga for a virtual summit of the Quadrilateral Framework (Quad) on Friday — the first time leaders of the Indo-Pacific grouping are meeting.

Q) Consider the following statements: 1. India’s indigenous COVID-19 vaccine Covaxin, from Hyderabad-based Bharat Biotech, has been taken off the clinical trial mode. 2. Covaxin is now authorized for emergency use on a par with Oxford-AstraZeneca’s Covishield

Which of the statements given above is/are correct?

A. 1 only B. 2 only C. Both 1 and 2 D. Neither 1 nor 2

Anc - C

India’s indigenous COVID-19 vaccine Covaxin, from Hyderabad-based Bharat Biotech, has been taken off the clinical trial mode and is now authorized for emergency use on a par with Oxford-AstraZeneca’s Covishield, NITI Aayog’s member (health) V.K. Paul said on Thursday.

43 TELEGRAM LINK: https://t.me/opdemy WEBSITE: www.opdemy.com

While congratulating the scientific community on the achievement, Dr. Paul cautioned that the COVID pandemic is not over and that people should not lower their guard.

Q) With reference to the National Capital Territory of Delhi Laws (Special Provisions) Second (Amendment) Bill, 2021, consider the following statements: 1. The 2011 Act provided for the regularisation of unauthorized colonies in the National Capital which existed as of 31st March 2002, and where construction took place till 1st June 2014. 2. The legislation provides that the unauthorized colonies which existed as of 1st June 2014 and have 50 percent development as of 1st January 2015, will be eligible for regularisation.

Which of the statements given above is/are correct?

A. 1 only B. 2 only C. Both 1 and 2 D. Neither 1 nor 2

Ans - D

The Parliament has passed the National Capital Territory of Delhi Laws (Special Provisions) Second (Amendment) Bill, 2021 with Lok Sabha approving it. Rajya Sabha had already passed the bill last month. The legislation aims to amend the National Capital Territory of Delhi Laws (Special Provisions) Second Act, 2011. The 2011 Act was valid till 31st December last year. The Bill seeks to extend this deadline to the end of December 2023. The 2011 Act provided for the regularisation of unauthorized colonies in the National Capital which existed as of 31st March 2002, and where construction took place till 1st June 2014. The Bill amends this to provide that unauthorized colonies will be identified for regularisation as per the National Capital Territory of Delhi (Recognition of Property Rights of Residents in Unauthorized Colonies) Act, 2019, and the National Capital Territory of Delhi (Recognition of Property Rights of Residents in Unauthorised Colonies) Regulations, 2019.

Q) A joint military exercise DUSTLIK II has commenced in Foreign Training Node Chaubatia, Ranikhet in Uttarakhand between India and which country?

A. The UK B. UAE C. Uzbekistan D. Afghanistan

Ans - C

The India – Uzbekistan joint military exercise DUSTLIK II has commenced in Foreign Training Node Chaubatia, Ranikhet in Uttarakhand. This is the Second Edition of the annual bilateral joint exercise of both armies. It will continue till the 19th of this month. The first edition of the exercise was held in Uzbekistan in November 2019. 45 Soldiers each from Uzbekistan and the Indian Army are participating in the exercise.

44 TELEGRAM LINK: https://t.me/opdemy WEBSITE: www.opdemy.com

Q) Which two countries have signed a Memorandum of Understanding (MoU) on behalf of their national governments, to jointly construct a lunar space station?

A. India and The US B. China and Russia C. China and Germany D. France and Germany

Ans B

The space agencies of China and Russia have signed a Memorandum of Understanding (MoU) on behalf of their national governments, to jointly construct a lunar space station that will be open to all countries. The MoU was officially signed between the Chinese space agency, China National Space Administration (CNSA) and the Russian space agency Roscosmos, on March 09, 2021.

Q) Which Bollywood actor will be honored with an award by the International Federation of Film Archives (FIAF)?

A. Amitabh Bachchan B. Nawazuddin Siddiqui C. Shah Rukh Khan D. Randeep Hooda

Ans - A

Megastar Amitabh Bachchan will be honored with an award by the International Federation of Film Archives (FIAF). The veteran actor will be the first Indian cinema personality to be bestowed with the FIAF Award for his dedication and contribution to the preservation of the world’s film heritage. Amitabh Bachchan was nominated for the award by the FIAF affiliate Film Heritage Foundation which is a not-for-profit organization founded by Shivendra Singh Dungarpur a filmmaker & archivist. This foundation is dedicated to preserve, restore, document, exhibit and study the film heritage of India.

Q) Who has recently been presented with the Baton of Honour and a letter of appreciation from President Ram Nath Kovind?

A. CDS Bipin Rawat B. Ajit Doval C. MN Naravane D. Kiran Bedi

Ans - D

President Ram Nath Kovind has presented a baton of honor and a letter of appreciation to former Puducherry lieutenant governor Kiran Bedi at Rashtrapati Bhavan. The former

45 TELEGRAM LINK: https://t.me/opdemy WEBSITE: www.opdemy.com

Puducherry lieutenant governor received a Baton of Honor decorated with the emblem of the Indian Republic and the appreciation letter recognizing her service in the Union Territory of Puducherry.

Q) ISRO develops Joint Earth Observation Satellite Mission Radar with the space agency of which country?

A. France B. Japan C. The US D. Russia

Ans - C

ISRO has completed the development of a Synthetic Aperture Radar (SAR) capable of producing extremely high-resolution images for a joint earth observation satellite mission with the US space agency NASA. NASA-ISRO SAR (NISAR) is a joint collaboration for a dual-frequency L and S-band SAR for earth observation.

Q) Which Bank has launched a range of wearable contactless payment devices under the brand Wear ‘N’ Pay?

A. State Bank of India B. Indian Overseas Bank C. Axis Bank D. ICICI Bank

Ans - C

Axis Bank has launched a range of wearable contactless payment devices under the brand Wear ‘N’ Pay. These devices come in different forms of wearables like a band, key chain, and loop and are available starting at Rs 750. The wearables are directly linked to the bank account of an Axis Bank customer and function like a regular debit card. It allows purchases at any merchant store which accepts contactless transactions.

Q) Herath festival is one of the biggest festivals of:

A. Kashmiri Pandits B. Lingayat Sect C. Dongria Kondh community D. None of the above

Ans - A

Prime Minister Narendra Modi greeted people on the occasion of the Herath festival. Herath festival is one of the biggest festivals of Kashmiri Pandits. The festival is marked by a night of praying followed by a day of feasting. Some say Herath means the night of Lord Shiva.

46 TELEGRAM LINK: https://t.me/opdemy WEBSITE: www.opdemy.com

Q) Consider the following statements. a) The Quadrilateral Security Dialogue is an informal strategic forum between Australia, India, Japan, and the United States b) QUAD was initiated as a dialogue in August 2007 by then-Prime Minister Shinzo Abe of Japan

Which of the above statements is/are correct?

A. 1 only B. 2 only C. Both 1 and 2 D. Neither 1 nor 2

Ans - C

Members of the Quadrilateral framework, or Quad, will become “closer than ever before”, Prime Minister Narendra Modi said on Friday, in his address to the first-ever leadership summit of the grouping. Addressing the virtual summit, Mr. Modi, President Joe Biden of the United States, Japanese Prime Minister Yoshihide Suga and Australian Prime Minister Scott Morrison highlighted cooperation among the member countries to beat the global pandemic, with the joint partnership on vaccines and emphasized the need for an “open” and “free” Indo-Pacific region.

Q) Consider the following statements: 1. The Supreme Court held that only independent persons should be appointed State Election Commissioners and not bureaucrats 2. The State Election Commission conducts all elections to theState, Panchayats and the Municipalities (Articles 243K, 243ZA).

Which of the statements given above is/are correct?

A. 1 only B. 2 only C. Both 1 and 2 D. Neither 1 nor 2

Anc - A

The Supreme Court on Friday held that independent persons and not bureaucrats should be appointed State Election Commissioners. A Bench, led by Justice Rohinton F. Nariman, in a judgment, said giving government employees the additional charge of State Election Commissioners is a “mockery of the Constitution”.

Q) Which of the following union ministries launched Mera Ration Mobile App for the benefit of those ration card holders who move to new places in search of livelihood?

47 TELEGRAM LINK: https://t.me/opdemy WEBSITE: www.opdemy.com

A. Ministry of Micro, Small and Medium Enterprises B. Ministry of Rural Development C. Ministry of Food Processing Industries D. Ministry of Consumer Affairs, Food, and Public Distribution

Ans - D

The Ministry of Consumer Affairs, Food, and Public Distribution launched the Mera Ration Mobile App for the benefit of those ration card holders who move to new places in search of livelihood. At present 32 States and Union Territories are covered under the One Nation One Ration Card (ONORC) and integration of the remaining four States and UTs are expected to be completed in the next few months. At present, the system covers nearly 69 Crore National Food Security Act- NFSA beneficiaries in the country. The ONORC scheme is being implemented by the Department of Food & Public Distribution under Ministry of Consumer Affairs, Food & Public Distribution for the nation-wide portability of ration cards under National Food Security Act (NFSA).

Q) IBM has signed an MoU with which state government to introduce STEM education for girls?

A. Maharashtra B. Uttar Pradesh C. Uttarakhand D. Madhya Pradesh

Ans - C

IBM has announced a collaboration with Samagra Shiksha Uttarakhand, the Government of Uttarakhand’s education mission, to introduce the ‘IBM STEM for Girls’ program in 130 secondary and higher secondary schools across five districts in the state. The collaboration is part of a three-year program between IBM and Uttarakhand State Government with the American India Foundation as the implementation partner to increase the participation of girls and women in STEM careers.

Q) Which of the following statements is/are correct? a) This year World Kidney Day is observed on 11th March 2021. b) The theme of the 2021 World Kidney day is “Living Well with Kidney Disease”.

A. Only a B. Only b C. Both D. None

Ans C

World Kidney Day is observed globally on the second Thursday of March every year. This year World Kidney Day is observed on 11th March 2021. The theme of the 2021 World Kidney day is

48 TELEGRAM LINK: https://t.me/opdemy WEBSITE: www.opdemy.com

“Living Well with Kidney Disease”. World Kidney Day is a global campaign aimed at raising awareness of the importance of our kidneys. World Kidney Day aims to raise awareness of the importance of our kidneys to our overall health and to reduce the frequency and impact of kidney disease and its associated health problems worldwide.

Q) World’s Most Powerful Supercomputer Fugaku is ready for use. It belongs to which country?

A. The US B. Japan C. South Korea D. China

Ans - B

The Japanese scientific research institute called RIKEN and Fujitsu started developing the “Fugaku” six years ago. It is the world’s most powerful supercomputers. Fugaku has been named after an alternative name for Mount Fuji. Now, this supercomputer is fully ready and developed in Japan and is now available for research use. This supercomputer has been developed with the aim of making the device core of the computing infrastructure of Japan.

Q) Which State/UT is celebrating the festival of Shivratri ‘Herath’?

A. Jammu & Kashmir B. Ladakh C. Gujarat D. Maharashtra

Ans - A

In the Union Territory of Jammu and Kashmir, the festival of Shivratri ‘Herath’ is being celebrated across Jammu and Kashmir. Festival Mahashivratri locally known as Herath in Kashmir is being celebrated across J&K by Kashmiri Pandit Community with religious fervor by offering “Watak Nath Pooja”.

Q) The Reserve Bank of India has removed which bank from its enhanced regulatory supervision or Prompt Corrective Action (PCA) framework?

A. Axis Bank B. HDFC Bank C. ICICI Bank D. Yes Bank

Ans - B

The Reserve Bank of India has removed IDBI Bank from its enhanced regulatory supervision or Prompt Corrective Action (PCA) framework. It was noted that as per published results for the

49 TELEGRAM LINK: https://t.me/opdemy WEBSITE: www.opdemy.com

quarter ending December 31, 2020, the bank is not in breach of the PCA parameters on regulatory capital, net NPA, and leverage ratio. The bank has also provided a written commitment that it would comply with the norms of minimum regulatory capital, net NPA, and leverage ratio on an ongoing basis and has apprised the RBI of the structural and systemic improvements that it has put in place which would help the bank in continuing to meet these commitments.

Q) Which of the following has ensured 100% compliance of payment of minimum wages to contract workers through e-application Shramik Kalyan Portal?

A. ISRO B. DRDO C. Reserve Bank of India D. Indian Railways

Ans - D

Indian Railway Shramik Kalyan e-application has been developed and launched on 1st October, 2018. E-Application ensures the compliance of provisions of the Minimum Wages Act and also ensures that contractual workers working in Indian Railways get their rightful due by enforcing the contractors to regularly upload wage payment data into e-application. This helps Railways as Principal Employer, to keep vigil over wages disbursed by contractors to contract workers. All PSUs working under the Ministry of Railways are also using this e-application.

Q) With reference to the State Election Commissioners, consider the following statements: 1. The Supreme Court held that independent persons and not bureaucrats should be appointed State Election Commissioners. 2. Article 241 deals with the Elections to the Panchayats.

Which of the statements given above is/are correct?

A. 1 only B. 2 only C. Both 1 and 2 D. Neither 1 nor 2

Ans - A

The Supreme Court held that independent persons and not bureaucrats should be appointed State Election Commissioners. It said that giving government employees the additional charge of State Election Commissioners is a “mockery of the Constitution”. The top court directed that the States should appoint independent persons as Election Commissioners all along the length and breadth of the country. The judgment criticized the Goa government for giving its Law Secretary the additional charge of State Election Commissioner.

50 TELEGRAM LINK: https://t.me/opdemy WEBSITE: www.opdemy.com

Q) Consider the following statements. a) The Centre will allow residents to fill the National Population Register (NPR) form on their own, through the online mode b) The details of the respondent will be displayed on a mobile application developed for conducting the Census exercise but no “biometrics or documents” will be collected.

Which of the above statements is/are correct?

A. 1 only B. 2 only C. Both 1 and 2 D. Neither 1 nor 2

Ans - C

The Centre will allow residents to fill the National Population Register (NPR) form on their own, through the online mode, a month before the door-to-door enumeration by Census officials starts. After filling the form online, residents will get a referral code that they can mention to the field enumerator at the time of her or his visit, according to a senior government official. The details of the respondent will be displayed on a mobile application developed for conducting the Census exercise but no “biometrics or documents” will be collected. These details will then be stored in the system.

Q) Consider the following statements: 1. The Archaeological Survey of India (ASI) on March 10 issued a notification for “reconstitution of the Advisory Committee for the Multidisciplinary Study of the River Sarasvati. 2. The Archaeological Survey of India is an Indian government agency attached to the Ministry of Culture

Which of the statements given above is/are correct?

A. 1 only B. 2 only C. Both 1 and 2 D. Neither 1 nor 2

Anc - C

The Centre has reconstituted an advisory committee to chalk out a plan for studying the mythical Saraswati river for the next two years after the earlier panel’s term ended in 2019. The Archaeological Survey of India (ASI) on March 10 issued a notification for “reconstitution of the Advisory Committee for the Multidisciplinary Study of the River Sarasvati”. The ASI had first set up the committee on December 28, 2017, for a period of two years.

51 TELEGRAM LINK: https://t.me/opdemy WEBSITE: www.opdemy.com

Q) With reference to the Seabuckthorn, consider the following statements: 1. It is a shrub that produces an orange-yellow colored edible berry. 2. It has been widely used for treating stomach, heart, and skin problems.

Which of the statements given above is/are correct?

A. 1 only B. 2 only C. Both 1 and 2 D. Neither 1 nor 2

Ans - C

The Himachal Pradesh government has decided to start planting sea buckthorn in the cold desert areas of the state this year. Seabuckthorn is a shrub that produces an orange-yellow colored edible berry. In India, it is found above the tree line in the Himalayan region, generally in dry areas such as the cold deserts of Ladakh and Spiti. In Himachal Pradesh, it is locally called chharma and grows in the wild in Lahaul and Spiti and parts of Kinnaur. Around 15,000 hectares in Himachal, Ladakh, Uttarakhand, Sikkim and Arunachal Pradesh are covered by this plant.

Q) Which state CM inaugurated the Swarnim International Shivratri Fair?

A. Uttar Pradesh B. Madhya Pradesh C. Haryana D. Himachal Pradesh

Ans - D

The famous Swarnim International Shivratri Fair has begun with Chief Minister Jai Ram Thakur formally launching it at the historic Paddal ground, Himachal Pradesh. Thousands of people dressed in their traditional attire participated in the procession carrying their local deities dancing all the way up to Paddal ground. Over 150 deities participated in the ‘Jaleb’, the traditional Shobha Yatra from almost all parts of the district. Earlier, the Chief Minister participated in the Ceremony and performed Puja at Shri Raj Madhav Rai Temple. Chief Minister felicitated the people the State on the auspicious occasion of International Shivratri festival, which was being celebrated this year as Swarnim International Shivratri Fair to mark glorious fifty years of Statehood.

Q) Which motor Company has launched ‘Wheels of Love’, a holistic programme that supports new parents in the workforce?

A. Tesla Motors B. Tata Motors C. Hyundai

52 TELEGRAM LINK: https://t.me/opdemy WEBSITE: www.opdemy.com

D. Hero Motor Corps

Ans - B

Tata Motors has launched ‘Wheels of Love’, a holistic programme that supports new parents in the workforce. This is a move to promote a progressive culture of care, inclusion and sensitization within the organisation across levels. A specially curated book, also aptly titled, Wheels of Love, puts forth various beliefs to enable new and expectant parents to successfully manage the needs of a growing family while also fulfilling their career goals. This guidebook also provides valuable insights for managers to support their team members as they progress through the various stages of parenthood.

Q) Who among the following has become the first Indian woman cricketer that has completed 10,000 international runs in all the cricket formats?

A. Mithali Raj B. Smriti Mandhana C. Harmanpreet Kaur D. Jhulan Goswami

Ans - A

The veteran Indian women’s cricketer Mithali Raj has become the first Indian woman cricketer that has completed 10,000 international runs in all the cricket formats. She also becomes the second woman cricketer at the international level to complete 10000 runs. She achieved the milestone of 10,000 after she hit a boundary off the Anne Bosch in 28th over of Indian innings in the ongoing third ODI match against South Africa.

Q) Which ministry has a portal called “Atmanirbhar Niveshak Mitra Portal” for information dissemination, hand-holding, and facilitation of domestic investors?

A. Ministry of Information & Broadcasting B. Ministry of Industry C. Ministry of Finance D. Ministry of Communications

Ans - B

The commerce and industry ministry has stated that the central government is working to develop a portal called “Atmanirbhar Niveshak Mitra Portal” for information dissemination, hand-holding, and facilitation of domestic investors. The portal Atmanirbhar Niveshak Mitra is currently is in the testing phase. It will be launched by May 1, 2021. The ministry is also working on a webpage which will be available in regional languages and mobile app.

Q) Which department/ministry has launched a new online certificate course on 5G technology?

A. Department of Telecommunication

53 TELEGRAM LINK: https://t.me/opdemy WEBSITE: www.opdemy.com

B. Ministry of Communication C. Ministry of I&B D. None of the above

Ans - A

The Department of Telecommunications (DoT) has launched a new online certificate course on 5G technology to be conducted by the National Telecommunications Institute for Policy Research, Innovation and Training (NTIPRIT), which is the training institute of DoT. The course, which will be conducted from 9 March 2021 onwards, is 36 hours long and will be conducted over a period of 12 weeks every Tuesday, Wednesday and Thursday from 3:30 pm to 4:30 pm. Participants will also be allowed to record the session if they happen to miss it.

Q) Which ministry has launched Mera Ration Mobile App for the benefit of those ration card holders who move to new places in search of livelihood?

A. Ministry of Corporate Affairs B. Ministry of Health & Family Affairs C. Ministry of Home Affairs D. Ministry of Consumer Affairs, Food & Public Distribution

Ans - D Ministry of Consumer Affairs, Food and Public Distribution launched Mera Ration Mobile App for the benefit of those ration card holders who move to new places in search of livelihood. At present 32 States and Union Territories are covered under One Nation One Ration Card (ONORC) and integration of the remaining four States and UTs are expected to be completed in the next few months. At present, the system covers nearly 69 Crore National Food Security Act- NFSA beneficiaries in the country. The ONORC scheme is being implemented by the Department of Food & Public Distribution under Ministry of Consumer Affairs, Food & Public Distribution for the nation- wide portability of ration cards under National Food Security Act (NFSA).

Q) The Zo people are an ethnic group of which of the following country?

1. India 2. Bangladesh 3. Myanmar

Select the correct answer using the code given below:

A. 1 only B. 1 and 2 only C. 2 and 3 only D. 1, 2 and 3

Ans - D

A Mizoram-based group representing the Zo indigenous people of India, Bangladesh and Myanmar has petitioned President Ram Nath Kovind and Prime Minister Narendra Modi to

54 TELEGRAM LINK: https://t.me/opdemy WEBSITE: www.opdemy.com

impose sanctions on military-ruled Myanmar. The Zo Reunification Organisation (ZORO) comprising the Chin-Kuki-Mizo-Zomi group of people have also asked the Centre not to turn away the Myanmar nationals who crossed over to escape the military regime and provide them shelter on humanitarian grounds. These four states – Arunachal Pradesh, Nagaland, Manipur, and Mizoram – share a 1,643 km border with Myanmar and people on either side are ethnically related.

Q) Consider the following statements. a) A pilot project launched in Kodagu entails installing bee boxes along the periphery of the forest and the villages to ward off elephant attacks. b) It is an initiative of the and Village Industries Commission (KVIC), Project RE-HAB (Reducing Elephant-Human Attacks using Bees) intends to create “bee fences”

Which of the above statements is/are correct?

A. 1 only B. 2 only C. Both 1 and 2 D. Neither 1 nor 2

Ans - C

A pilot project launched in Kodagu entails installing bee boxes along the periphery of the forest and the villages with the belief that the elephants will not venture anywhere close to the bees and thus avoid transgressing into human landscape. This idea stems from the elephants’ proven fear of the bees. An initiative of the Khadi and Village Industries Commission (KVIC), Project RE-HAB (Reducing Elephant-Human Attacks using Bees) intends to create “bee fences” to thwart elephant attacks in human habitations using honeybees. The pilot project was launched at four locations around Chelur village in Kodagu district by KVIC chairman Vinai Kumar Saxena on Monday.

Q) Consider the following statements: 1. In the elite Indian Institutes of Management (IIMs), more than 60% of SC and OBC reserved positions are vacant 2. Both the IITs and the IIMs have been lobbying for an exemption from such faculty quota requirements.

Which of the statements given above is/are correct?

A. 1 only B. 2 only C. Both 1 and 2 D. Neither 1 nor 2

Anc - C

55 TELEGRAM LINK: https://t.me/opdemy WEBSITE: www.opdemy.com

More than half of the faculty positions reserved for the OBCs in Central institutions of higher education are vacant while about 40% of those reserved for the Scheduled Castes and Tribes also remain unfilled. The situation is particularly acute in the elite Indian Institutes of Management (IIMs), where more than 60% of SC and OBC reserved positions are vacant, while almost 80% of positions reserved for the STs have not been filled. This means that out of 24 positions reserved for the STs, only five have been filled. For the Indian Institutes of Technology (IITs), data has only been provided for non-faculty positions. Both the IITs and the IIMs have been lobbying for exemption from such faculty quota requirements.

Q) With reference to the Commission for Air Quality Management, consider the following statements: 1. It is a statutory body. 2. The commission is headed by the Minister of Petroleum and Natural Gas.

Which of the statements given above is/are correct?

1 only 2 only Both 1 and 2 Neither 1 nor 2

Ans - D

The Center has dissolved the Commission for Air Quality Management. The commission was headed by M.M. Kutty, a former Secretary in the Ministry of Petroleum and Natural Gas. The dissolution happened despite the nodal Union Environment Ministry submitting the paperwork to the Union Cabinet Secretariat, required to give legal backing to the commission. The body came into being in last October on the back of an ordinance — a temporary measure — and the law requires that a formal Bill be presented to Parliament within six weeks of it reconvening — in this case — January 29 when the Budget Session began. Before a Bill is tabled in Parliament it needs to be approved by the Union Cabinet.

56 TELEGRAM LINK: https://t.me/opdemy WEBSITE: www.opdemy.com

UPSC CSE PRELIMS 2021

CRASH COURSE DATE SUBJECT Budget and Economic 10/04/2021, 11/04/2021 Survey 12/04/2021, 13/04/2021 Art and Culture 14/0/2021, 15/04/2021, Economics 16/04/2021 17/18/2021,18/04/2021, Modern Indian History 19/04/2021 20/04/2021,21/04/2021, Geography 22/04/2021 23/04/2021,24/04/2021, Environment 25/04/2021 26/04/2021,27/04/2021, 28/04/2021,29/04/2021, Polity 30/04/2021 01/05/2021, 02/05/2021 Science and Tech *Note: Also, there will be Lectures on CSAT as and when time permits* PRICE: 1,999 For Any Queries: 7988797680, 7999136838, 6295118940

57 TELEGRAM LINK: https://t.me/opdemy WEBSITE: www.opdemy.com

Q) Indian Navy Ship Jalashwa arrived at the port of Anjouan in which country with one thousand metric tonnes of rice, highlighting the exemplary ties between India and the country within the framework of PM Modi’s vision of SAGAR ( Security and Growth for all in the Indian Ocean Region)?

A. Seychelles B. Madagascar C. Comoros D. Mozambique

Ans - C

Indian Navy Ship Jalashwa arrived at the port of Anjouan in Comoros with one thousand metric tonnes of rice. This highlights the exemplary ties between India and Comoros within the framework of PM Modi’s vision of SAGAR ( Security and Growth for all in the Indian Ocean Region). Comoros is an island country in the Indian Ocean.

Q) World Consumer Rights Day is celebrated on -

A. 14th March B. 15th March C. 16th March D. 17th March

Ans - B

It is celebrated on 15 March across the world annually to spread awareness about the rights of consumers and their needs. A special message was sent by President John F Kennedy to the US Congress on 15 March, 1962 and so the day was inspired by him. In the message, he formally addressed the issue of consumer rights. To do so, he was the first leader.

Q) The National Institute of Oceanography (NIO) will launch the first-of-its-kind project of Genome Mapping in which ocean?

A. Atlantic B. Pacific C. Arctic D. Indian

Ans - D

In the Indian Ocean, the National Institute of Oceanography (NIO) will launch the first-of-its-kind project of Genome Mapping. The Indian Ocean covers about 20% of the Earth's water surface and so is the third largest the water body in the world.

58 TELEGRAM LINK: https://t.me/opdemy WEBSITE: www.opdemy.com

Q) AEG12 strongly inhibits the family of viruses that cause yellow fever, dengue, West Nile, and Zika, and also weakly inhibits coronaviruses, according to scientists at the US National Institutes of Health (NIH) and their collaborators. What is it?

A. Cow protein B. Pork protein C. Mosquito protein D. Bat protein

And - C

The researchers found that AEG12 works by destabilizing the viral envelope, breaking its protective covering. The protein does not affect viruses that do not have an envelope. At the molecular level, AEG12 rips out the lipids (the fat-like portions of the membrane that hold the virus together). The findings, however, could lead to therapeutics against viruses that affect millions of people around the world.

Q) Center has dissolved the Commission for Air Quality Management. The commission was headed by M.M. Kutty, a former Secretary in which ministry?

A. Ministry of Environment B. Ministry of Education C. Ministry of Petroleum and Natural Gas D. Ministry of Corporate Affairs

Ans - C

The commission was headed by M.M. Kutty, a former Secretary in the Ministry of Petroleum and Natural Gas. The dissolution happened despite the nodal Union Environment Ministry submitting the paper work to the Union Cabinet Secretariat, required to give legal backing to the commission.

Q) The Centre has reconstituted an advisory committee to chalk out a plan for studying which mythical river for the next two years, after the earlier panel’s term, ended in 2019?

A. Saraswati B. Ganga C. Yamuna D. Brahmaputra

Ans - A

The Archaeological Survey of India (ASI) on March 10 issued a notification for “reconstitution of the Advisory Committee for the Multidisciplinary Study of the River Sarasvati”. The ASI had first set up the committee on December 28, 2017, for a period of two years. The committee would continue to be chaired by the Culture Minister.

59 TELEGRAM LINK: https://t.me/opdemy WEBSITE: www.opdemy.com

Q) To commemorate the heroism of Lachit Borphukan on 24 November each year Lachit Divas (Lachit Day) is celebrated in which of the following state?

A. Sikkim B. Nagaland C. Himachal Pradesh D. Assam

Ans - D

Opposition parties in poll-bound Assam slammed Prime Minister Narendra Modi for referring to 17th-century Ahom general Lachit Borphukan as a freedom fighter. Lachit Borphukan (1622 - 1672) was a commander and Borphukan (Phu-Kon-Lung) in the Ahom kingdom, located in present-day Assam. He is known for his leadership in the 1671 Battle of Saraighat that thwarted a drawn-out attempt by Mughal forces under the command of Ramsingh I to take over Ahom kingdom.

Q) The Congress in Assam has tweaked Bhaona for a political statement against the Citizenship(Amendment) Act and National Register of Citizens. Bhaona is a traditional form of entertainment, with religious messages, prevalent in which state/UT?

A. Mizoram B. Assam C. Pondicherry D. Kerala

Ans - B

Bhaona is a traditional form of entertainment, with religious messages, prevalent in Assam. It is a creation of Sankardeva, written in the early sixteenth century. The plays of bhaona are popularly known as Ankiya Nats and their staging is known as bhaona. The bhaonas are written in the Assamese and Brajavali languages.

Q) According to a report by SIPRI, India's arms import has dropped to

A. 23% B. 33% C. 43% D. 50%

Ans - B

Arms imports decreased by 33% between 2011–15 and 2016–20 while India continues to remain the second-largest arms importer after Saudi Arabia, according to a report from Swedish think tank Stockholm International Peace Research Institute (SIPRI). The overall drop in arms imports between 2011–15 and 2016–20 was due to complex and lengthy procurement

60 TELEGRAM LINK: https://t.me/opdemy WEBSITE: www.opdemy.com

processes, combined with attempts to reduce its dependence on Russian arms by diversifying its network of arms suppliers. Russia was the largest arms supplier in both years. However, Russia’s deliveries dropped by 53% between the two periods and its share of Indian arms imports fell from 70 to 49%. France and Israel were the second and third largest arms suppliers in 2016–20. India’s arms imports from France increased by 709% while those from Israel rose by 82%. The U.S. was the fourth-largest supplier in 2016–20. Combat aircraft and associated missiles made up more than 50% of arms imports.

Q) For the first time ever, the Border Roads Organisation (BRO) has started work on reopening the crucial Baralacha Pass in which state/UT?

A. Ladakh B. Sikkim C. Arunachal Pradesh D. Himachal Pradesh

Ans - D

For the first time ever, the Border Roads Organisation (BRO) has started work on reopening the crucial Baralacha Pass in Himachal Pradesh much before schedule to restore connectivity to Leh in Ladakh. Bara-lacha la also known as Bara-lacha Pass is a high mountain pass in Zanskar range, connecting Lahaul district in Himachal Pradesh to Leh district in Ladakh, situated along the Leh–Manali Highway. The pass also acts as a water divide between the Bhaga river and the Yunam river. The Bhaga river, a tributary of the Chenab river, originates from Surya taal lake, which is situated a few of kilometers from the pass towards Manali.

Q) The National Vaccination Day, (also called the National Immunization Day), is celebrated every year on -

A. 16th March B. 17th March C. 18th March D. 19th March

Ans - A

The National Vaccination Day, (also called the National Immunization Day), is celebrated every year on March 16 in India, to convey the importance of vaccination to the entire nation. The prime purpose behind the celebration of National Vaccination Day is to make all people aware of arming against polio and eradicating it completely from the world. In 2021, the National Vaccination Day is important as the country has started its biggest Covid-19 immunization programme. The day was first observed in the year 1995. It was in 1995 that India started the Pulse Polio Programme and the first dose of Oral Polio Vaccine was given. According to the World Health Organization, immunization is a process through which an individual’s immune system becomes fortified against foreign harm causing agent.

61 TELEGRAM LINK: https://t.me/opdemy WEBSITE: www.opdemy.com

Q) Which ministry has announced a new scheme for tourist vehicle operators, under which any tourist vehicle operator may apply for an “All India tourist authorization and permit” through online mode?

A. Ministry of Tourism B. Ministry of Corporate Affairs C. Ministry of Road Transport and Highways D. Ministry of Home Affairs

Ans - C

The Ministry of Road Transport and Highways has announced a new scheme for tourist vehicle operators, under which any tourist vehicle operator may apply for an “All India tourist authorization and permit” through online mode. The new set of rules will be known as “All India Tourist Vehicles Authorization and Permit Rules, 2021”. This will come into effect from April 01, 2021. The new rules for permits are aimed at promoting tourism across the states in our country, while simultaneously, helping to grow the revenue of state Governments. The permits will be issued, after relevant documents are submitted and fees deposited, within 30 days of submission of such applications. The authorization/permit will be granted, for a period of three months or its multiples thereof, not exceeding three years at a time.

Q) India’s foreign-exchange reserves recently surpassed which country to become the world’s fourth-largest reserve?

A. Japan B. Russia C. China D. Switzerland

Ans - B

India’s foreign-exchange reserves surpassed Russia’s to become the world’s fourth-largest reserve. The foreign currency holdings of India stood at $580.3 billion as of March 5, as per the data by the Reserve Bank of India (RBI). The reserve of Russia was $580.1 billion. Overall, China has the largest reserves, followed by Japan and Switzerland, as on the International Monetary Fund table. Reserves for both countries have mostly flattened out this year after months of rapid increase. India pulled ahead as Russian holdings declined at a faster rate in recent weeks. India’s reserves, enough to cover roughly 18 months of imports, have been bolstered by a rare current-account surplus, rising inflows into the local stock market, and foreign direct investment.

Q) Which State has announced the launch of SAAMAR (Strategic Action for Alleviation of Malnutrition and Anemia Reduction) campaign to tackle malnutrition in the state?

A. Orissa Govt B. Jharkhand Govt

62 TELEGRAM LINK: https://t.me/opdemy WEBSITE: www.opdemy.com

C. Bihar Govt D. Andhra Pradesh Govt

Ans - B

The campaign aims to identify anemic women and malnourished children and converge various departments to effectively deal with the problem in a state where malnutrition has been a major problem. AAMAR has been launched with a 1000 days target, under which annual surveys will be conducted to track the progress. To tackle severe acute malnutrition children, every Anganwadi Centres will be engaged to identify these children and subsequently will be treated at the Malnutrition Treatment Centers. In the same process, the anemic women will also be listed and will be referred to health centers in serious cases. All of these will be done by measuring Mid-Upper Arm Circumference (MUAC) of women and children through MUAC tapes and Edema levels, swelling in a small area or the entire body—malnutrition is one of the reasons attributed to this disease. Angawadi’s Sahayia and Sevika will take them to the nearest Health Centre where they will be checked again and then registered on the portal of State Nutrition Mission.

Q) World Oral Health Day is observed on -

A. 19th March B. 20th March C. 21st March D. 22nd March

Ans - B

World Oral Health Day is observed every year on 20th March. It focuses on highlighting the benefits of good oral health, spreads awareness about oral diseases, and promotes maintenance of oral hygiene. The theme for the next three years, 2021-2023 is: Be Proud Of Your Mouth. The day is an initiative of FDI World Dental Federation; an organization that brings together the world of dentistry with the aim of achieving optimal oral health for everyone.

Q) International Day of Happiness is observed on -

A. 20th March B. 21st March C. 22nd March D. 23rd March

Ans - A

International Day of Happiness is celebrated on March 20 to promote happiness as a fundamental human right for all human beings across the world. The 2021 International Day the Happiness campaign theme is ‘Keep Calm. Stay Wise. Be Kind’. Since 2013, the United

63 TELEGRAM LINK: https://t.me/opdemy WEBSITE: www.opdemy.com

Nations have celebrated the International Day of Happiness as a way to recognize the importance of happiness in the lives of people around the world. The day was proclaimed by the United Nations General Assembly on 28 June 2012.

Q) Which ministry has launched the GRAM UJALA programme in Arrah, Bihar?

A. Ministry of Urban Development B. Ministry of Power C. Ministry of Rural Development D. Ministry of Corporate Affairs

Ans - B

Under the programme, 7 watt and 12-Watt LED bulbs with 3 years warranty will be given to rural consumers against submission of working Incandescent bulbs. LEDs will be available for only Rs 10 each for each household, in exchange for working condition old incandescent lamps. Each household will get up to 5 LEDs. In the first phase of this programme, 15 million (1.5 crores) LED bulbs will be distributed across villages of Aarah (Bihar), Varanasi (Uttar Pradesh), Vijaywada (Andhra Pradesh), Nagpur (Maharashtra), and village in western Gujarat. The Gram Ujala programme will be implemented in villages of the 5 districts only. These rural households will also have meters installed in their houses to account for usage.

Q) decorative jaapis (field ), hand-woven and bell-metal xorais are in news these days. They belong to which state?

A. Tamil Nadu B. Kerala C. West Bengal D. Assam

Ans - D

Jaapi: The is a conical made of bamboo and covered with dried tokou (a palm tree found in rainforests of Upper Assam) leaves. Today, the bulk of Assam’s jaapis are made by artisans based in a cluster of villages in Nalbari district. : The Gamosa, which literally translates to a cloth to wipe one’s body, is omnipresent in Assam, with wide-ranging uses. It can be used at home as a towel (uka gamosa) or in public functions (phulam/floral gamosa) to felicitate dignitaries or celebrities. Xorai - Made of bell-metal, the xorai — essentially a tray with a stand at the bottom, with or without a cover — can be found in every Assamese household.

Q) Consider the following statements. a) India abstained from a crucial vote on Sri Lanka’s rights record at the United Nations Human Rights Council in Geneva. b) The resolution on ‘Promoting reconciliation, accountability and human rights in Sri Lanka’ is about evidence of war crimes in Sri Lanka, committed by the armed forces and the LTTE.

64 TELEGRAM LINK: https://t.me/opdemy WEBSITE: www.opdemy.com

Which of the above statements is/are correct?

A. 1 only B. 2 only C. Both 1 and 2 D. Neither 1 nor 2

Ans - C

India on Tuesday abstained from a crucial vote on Sri Lanka’s rights record at the United Nations Human Rights Council in Geneva. The resolution on ‘Promoting reconciliation, accountability and human rights in Sri Lanka’ was, however, adopted after 22 states of the 47-member Council voted in its favor. Sri Lanka, which had earlier deemed the resolution “politically motivated”, was quick to reject the UN move to collect and preserve evidence of war crimes in the country, committed by the armed forces and the LTTE.

Q) Consider the following statements: 1. Citizens above the age of 45, irrespective of comorbidities, will be eligible for COVID-19 vaccination from April 1 2. At present, only citizens above 60 and those over 45 with co-morbidities are allowed to get the vaccination.

Which of the statements given above is/are correct?

A. 1 only B. 2 only C. Both 1 and 2 D. Neither 1 nor 2

Anc - C

Citizens above the age of 45, irrespective of comorbidities, will be eligible for COVID-19 vaccination from April 1, the government announced on Tuesday. At present, only citizens above 60 and those over 45 with co-morbidities are allowed to get the vaccination. “This decision has been taken by the Union Cabinet on the basis of the advice of the COVID task force and experts” said Information and Broadcasting Minister Prakash Javadekar

Q) Buldhana district is famous for water conservation. It is located in:

A. Madhya Pradesh B. Maharashtra C. Assam D.

Ans - A

65 TELEGRAM LINK: https://t.me/opdemy WEBSITE: www.opdemy.com

Minister for Jal Shakti informed Rajya Sabha about the 'Buldhana Pattern' for water conservation. NITI Aayog has prepared draft guidelines for convergence of the activities of - (i) sourcing earth for improvement/construction of national highways, state roads and railways and (ii) water conservation works through de-siltation/ deepening of water bodies. This two-fold activity has already been successfully implemented as a pilot project in the Buldhana district of Maharashtra state.

Q) Which of the following statements is/are true regarding the recently-released study by defense website Military Direct? a) India has the 4th strongest military. b) China has the strongest military in the world at rank 1.

A. Only a B. Only b C. Both D. None

Ans - C

China has the strongest military force in the world while India stands at number four, according to a study released by the defense website Military Direct. China has the strongest military in the world, scoring 82 out of 100 points in the index. The USA, despite their enormous military budgets, comes in 2nd place with 74 points, followed by Russia with 69, India at 61 and then France with 58. The U.K. just about makes the Top 10, coming in 9th place with a score of 43.

Q) The Gandhi Peace Prize for the year 2019 is being conferred on (Late) His Majesty Sultan Qaboos bin Said Al Said of -

A. Yemen B. Oman C. Saudi Arabia D. Qatar

Ans - B

The jury for Gandhi Peace Prize is chaired by Prime Minister Narendra Modi and comprises of two ex-officio members, namely the chief justice of India and leader of the single largest opposition party in the Lok Sabha. Two eminent members are also part of the jury -- Om Birla, Speaker of the Lok Sabha, and Bindeshwar Pathak, Founder of Sulabh International Social Service Organisation.

Q) Farmers of which state has started the fresh fruit cake ‘movement’?

A. Odisha

66 TELEGRAM LINK: https://t.me/opdemy WEBSITE: www.opdemy.com

B. Tamil Nadu C. Gujarat D. Maharashtra

Ans - D

Fruit growers in rural Maharashtra have started an innovative ‘movement’ to promote fresh fruit cakes as a healthier option, instead of the traditional bakery-made cakes. According to farmers and agrarian experts, the aim of this ‘spontaneous’ movement is to encourage farmers and their families to increase the intake of fruits in their diet and to find a new way of selling their produce in times of the pandemic.

Q) World Meteorological Day observed globally on -

A. 21st March B. 22nd March C. 23rd March D. 24th March

Ans - C

The World Meteorological Day is celebrated every year on 23 March to commemorate the date of the establishment of the World Meteorological Organization on 23 March 1950. The day also highlights the contribution that the National Meteorological and Hydrological Services make to the safety and well-being of society. The World Meteorological Day theme is “The ocean, our climate and weather” which celebrates WMO’s focus on connecting the ocean, climate, and weather within the Earth System.

Q) In the 67th National Film Award, which state has been awarded for being Most Film Friendly State?

A. Maharashtra B.Uttar Pradesh C.Sikkim D. Punjab

Ans - C

Sikkim has bagged the national award as the ‘Most Film Friendly State’ recording the emergence of the Himalayan State as a new film shooting landscape.

Q) In the 67th National Film Award, which two actors shared the best actor award?

A. Manoj Bajpayee and Dhanush B. Manoj Bajpayee and Nawazuddin Siddiqui C. Amitabh Bachchan and Ayushmaan Khurana D. Ayushmaan Khurana and Rajkumar Rao

67 TELEGRAM LINK: https://t.me/opdemy WEBSITE: www.opdemy.com

Ans - A

Manoj Bajpayee and Dhanush shared the best actor award. While Manoj was awarded for his measured performance as a retired police constable in Bhonsle, Dhanush won it for depicting a range of emotions in Vetrimaaran’s multi-layered revenge drama Asuran.

Q) With reference to the Ganga Quest 2021, consider the following statements: 1. Ganga Quest is an international bilingual (Hindi and English) online quiz on Ganga. 2. It is being organized by NITI Aayog.

Which of the statements given above is/are correct?

1 only 2 only Both 1 and 2 Neither 1 nor 2

Ans - A

Ganga Quest 2021, an online competitive quiz was launched. Ganga Quest is an international bilingual (Hindi and English) online quiz on Ganga. It is being organized by National Mission for Clean Ganga (NMCG), an authority under the Government of India which implements the ‘Namami Gange’ programme and TREE Craze Foundation (TCF), a not-for-profit organization committed to Ganga, Rivers, and Environment.

Q) Consider the following statements. a) Chief Justice of India Sharad A. Bobde has recommended Justice N.V. Ramana as the next Chief Justice of India. b) Justice N.V. Ramana set to take over as 47th CJI

Which of the above statements is/are correct?

A. 1 only B. 2 only C. Both 1 and 2 D. Neither 1 nor 2

Ans - A

Justice N.V. Ramana set to take over as 48th CJI Chief Justice Bobde makes a formal recommendation to govt. Justice Ramana is now set to take over as the 48th Chief Justice of India from April 24. Chief Justice Bobde handed over a copy of his letter of recommendation to Justice Ramana on Wednesday after sending it to the government. The Centre had recently asked Chief Justice Bobde, who is retiring on April 23, to initiate the transition process to the top judicial office. Justice Ramana will be the CJI till August 26, 2022.

68 TELEGRAM LINK: https://t.me/opdemy WEBSITE: www.opdemy.com

Q) Consider the following statements: 1. Government of National Capital Territory of Delhi (Amendment) Bill, 2021 aims to "further define the responsibilities of the elected government and Lieutenant Governor (LG) in Delhi". 2. According to the bill, the expression 'Government' referred to in any law to be made by the Legislative Assembly shall mean the Lieutenant Governor (LG).

Which of the statements given above is/are correct?

A. 1 only B. 2 only C. Both 1 and 2 D. Neither 1 nor 2

Anc - C

The Rajya Sabha on Wednesday passed the Government of National Capital Territory of Delhi (Amendment) Bill that seeks to empower the Lieutenant-Governor in Delhi. The Opposition, including the Aam Aadmi Party and the Congress, walked out just before the Bill was put to vote as a mark of protest. Earlier, the Biju Janata Dal, the Samajwadi Party, and the YSR Congress had walked out. Recently, the central government introduced the Government of National Capital Territory of Delhi (Amendment) Bill, 2021 in the Lok Sabha to amend the Government of National Capital Territory of Delhi Act, 1991. It aims to "further define the responsibilities of the elected government and Lieutenant Governor (LG) in Delhi".

Q) #FreedomPineapple campaign recently seen in the news is in the context of which country?

A. Taiwan B. India C. France D. U.S.A

Ans - A

Ties between China and Taiwan, which have historically been rocky over issues such as sovereignty, foreign relations, and military build-up, are now being tested by pineapples. On March 1, China banned the import of pineapples from Taiwan, alleging there was a risk of “harmful creatures” that could threaten its own agriculture. Since then, an angry Taiwan has refuted China’s claims of pests being found in imported pineapples and has gone on to insist that the move is aimed at increasing political pressure on Taiwan, which China considers its own province. Following this, Taiwanese President Tsai Ing-wen kicked off a “pineapple challenge” on social media to attract more Taiwanese consumers to buy the fruit and counter China’s move.

69 TELEGRAM LINK: https://t.me/opdemy WEBSITE: www.opdemy.com

Taiwan’s foreign minister also urged “like-minded friends around the globe to stand with #Taiwan & rally behind the #FreedomPineapple”.

Q) India and ___ have agreed to recognize each other’s offices to act mutually as competent International Searching and International Preliminary Examining Authority (ISA/IPEA) for any international patent application filed with them?

A. The US B. Japan C. France D. Russia

Ans - B

India and Japan have agreed to recognize each other’s offices to act mutually as competent International Searching and International Preliminary Examining Authority (ISA/IPEA) for any international patent application filed with them. This memorandum of cooperation on the industrial property was arrived at during the 4th review meeting between DPIIT and Japan Patent Office. India had approved an MoC between the Indian Patent Office (IPO) and the JPO for entering into a PPH on a pilot basis for three years. This is a reciprocal arrangement, so the JPO will also be able to accelerate the prosecution of a Japanese application using positive examination results received from the IPO. The IPO will only offer acceleration for patent applications in certain technical fields such as physics, computer science, IT, electronics, metallurgy, and automobiles.

Q) India is conducting the Pabbi-Anti-Terror 2021 with which two countries?

A. France & Israel B. The US & The UK C. Pakistan & China D. Bangladesh & Indonesia

Ans - C

The eight-member bloc said that the decision to hold the joint exercise ‘Pabbi-Anti-Terror 2021’ was announced during the 36th meeting of the Council of the Regional Anti-Terrorist Structure (RATS) held in Tashkent, Uzbekistan. Members of the Shanghai Cooperation Organisation, including India, Pakistan, and China, will hold a joint anti-terrorism exercise this year. “Decisions have been made to improve cooperation between the competent authorities of the SCO member states in identifying and suppressing channels that finance terrorist activities. Delegations of the competent authorities of India, Kazakhstan, China, the Kyrgyz Republic,

Pakistan, Russia, Tajikistan, Uzbekistan, and the RATS Executive Committee attended the meeting.

70 TELEGRAM LINK: https://t.me/opdemy WEBSITE: www.opdemy.com

Q) Which state has decided to set up “India’s first government-run ambulance network” for animals?

A. Kerala B. Uttar Pradesh C. Madhya Pradesh D. Andhra Pradesh

Ans - D

The state government of Andhra Pradesh has decided to set up “India’s first government-run ambulance network” for animals. This decision was taken in order to further boost the animal husbandry and veterinary sector in the state. Ambulance network as one of its main missions in order to help in reaching out to the distressed animals to provide them with proper animal healthcare. The Animal Husbandry Department was directed to set up one Mobile Ambulance Veterinary Clinic at each of the assembly constituencies. A total of 175 mobile ambulances (veterinary) clinics will be placed at Assembly Constituency Level on the lines of 108 Services for providing Veterinary services at the doorstep.

Q) ISRO successfully demonstrated free-space Quantum Communication over a distance of how many meters?

A. 100 m B. 200 m C. 300 m D. 400 m

Ans - C

In a first-of-its-kind initiative, the Indian Space Research Organisation (ISRO) successfully demonstrated free-space Quantum Communication over a distance of 300 meters, with several technologies developed within the country. The demonstration included live video conferencing using quantum-key-encrypted signals, at Space Applications Centre (SAC), Ahmedabad, between two line-of-sight buildings within the campus.

The experiment was done at night to ensure that there is no interference from the direct sunlight. This is a major milestone achievement for unconditionally secured satellite data communication using quantum technologies.

Q) World Tuberculosis Day is observed every year on -

A. 23rd March B. 24th March C. 25th March D. 26th March

Ans - B

71 TELEGRAM LINK: https://t.me/opdemy WEBSITE: www.opdemy.com

World Tuberculosis Day is observed every year on 24 March to create awareness among the public about the global epidemic of tuberculosis (TB) and efforts to eliminate the disease. The date marks the day in 1882 when Dr. Robert Koch announced that he had discovered the bacterium that causes TB, which opened the way towards diagnosing and curing this disease. The theme of World TB Day 2021 – ‘The Clock is Ticking’ –conveys the sense that the world is running out of time to act on the commitments to end TB made by global leaders. This is especially critical in the context of the COVID-19 pandemic that has put End TB progress at risk, and to ensure equitable access to prevention and care in line with WHO’s drive towards achieving Universal Health Coverage.

Q) India Legends, captained by Sachin Tendulkar, defeated which country team by 14 runs in the final of Road Safety World Series to clinch the title at Shaheed Veer Narayan Singh International Stadium in Raipur, Chhattisgarh?

A. Sri Lanka Legends B. England Legends C. South Africa Legends D. Bangladesh Legends

Ans - A

In Cricket, India Legends defeated Sri Lanka Legends by 14 runs in the final of Road Safety World Series to clinch the title at Shaheed Veer Narayan Singh International Stadium in Raipur, Chhattisgarh, tonight. Yusuf Pathan who smashed an unbeaten 62 and picked up two wickets in his four-over spell was adjudged player of the match for his all-round performance in the summit clash. Sachin Tendulkar, captain of the India Legends, finished the tournament as the third-highest run-getter in the tournament. He scored 234 runs in the tournament. India Legends, after being sent in to bat, put on a huge total of 181 for four from their allotted 20 overs. Yuvraj Singh continued his good form with the bat and scored 60 runs off just 41 balls. Sri Lanka Legends made 167 for 7 in the allotted 20 overs. For India Legends, Yusuf Pathan took 2 wickets for 26 runs while Irfan Pathan took two wickets for 29 runs.

Q) With reference to the Permanent Indus Commission, consider the following statements: 1. The Permanent Indus Commission (PIC) is a bilateral commission consisting of officials from India and Pakistan. 2. It was created to implement and manage the goals and objectives and outlines of the IndusWaters Treaty (IWT) which was signed in September 1960 with the United Nations standing guarantee for any dispute resolution.

Which of the statements given above is/are correct?

1 only 2 only Both 1 and 2 Neither 1 nor 2

Ans - A

72 TELEGRAM LINK: https://t.me/opdemy WEBSITE: www.opdemy.com

After a gap of more than two and a half years Indian and Pakistani delegations began the 116th Meeting of the Permanent Indus Commission. The Permanent Indus Commission (PIC) is a bilateral commission consisting of officials from India and Pakistan. It was created to implement and manage the goals and objectives and outlines of the Indus Waters Treaty (IWT) which was signed in September 1960 with World Bank standing guarantee for any dispute resolution. The last meeting of the Permanent Indus Commission was held on August 29, 2018. The meeting which coincided with the National Day of Pakistan is being viewed as part of the broader process of normalization of bilateral ties between the two neighbors.

Q) With reference to the Collegium System, consider the following statements. a) Appointment and transfer of judges is not done by an Act of Parliament or by a provision of the Constitution. b) The outgoing CJI recommends his successor consulting the Collegium, which sends the recommendation to the Law Minister, who forwards it to the PM to advise the President.

Which of the above statements is/are correct?

A. 1 only B. 2 only C. Both 1 and 2 D. Neither 1 nor 2

Ans - C

The CJI consults the rest of the Collegium members, as well as the senior-most judge of the court hailing from the High Court to which the recommended person belongs. The Collegium sends the recommendation to the Law Minister, who forwards it to the Prime Minister to advise the President. The President of India appoints the CJI and the other SC judges. As far as the CJI is concerned, the outgoing CJI recommends his successor.

Q) Consider the following statements: 1. Postal voting for electors aged above 80 is underway in various Assembly constituencies of Tamil Nadu for the first time. 2. Earlier, only service voters and those drafted for election work were allowed the facility.

Which of the statements given above is/are correct?

A. 1 only B. 2 only C. Both 1 and 2 D. Neither 1 nor 2

Anc - C

73 TELEGRAM LINK: https://t.me/opdemy WEBSITE: www.opdemy.com

Postal voting for electors aged above 80 is underway in various Assembly constituencies of Tamil Nadu. As of March 22, 12-D forms (seeking postal ballots) had been received by Returning Officers from over 1.49 lakh elderly electors. This is the first time certain sections of the public in the State have been allowed to cast postal ballots. Earlier, only service voters and those drafted for election work were allowed the facility.

Q) Zabarwan range recently seen in news is located in:

A. Jammu and Kashmir B. Kerala C. Andhra Pradesh D. Gujarat

Ans - A

Prime Minister Narendra Modi asked everyone to visit the tulip garden, located at the foothills of the Zabarwan range in Jammu and Kashmir. The tulip garden will be thrown open to the public on March 25, 2021. The garden will see over 15 lakh flowers of more than 64 varieties in bloom. The Zabarwan Range is a short sub-mountain range between Pir Panjal and the Great Himalayan Range in the central part of the Kashmir Valley in the Union Territory of Jammu and Kashmir in India.

Q) Which state's tuberculosis eradication programme had won national honor as the state bagged the Union government’s award for reducing the prevalence of the disease through various systematic initiatives?

A. Tamil Nadu B. Kerala C. Maharashtra D. Karnataka

Ans - B

Kerala’s tuberculosis eradication programme had won national honor as the state bagged the Union government’s award for reducing the prevalence of the disease through various systematic initiatives. Kerala is the only state chosen for recognition in the state category. The southern state has reduced the TB prevalence rate by 5 percent in the last five years as part of achieving sustainable development goals. The nation has set an ambitious goal to achieve “End Tuberculosis” by reducing the incidence of new TB cases by 80% by 2025, five years ahead of UN Sustainable Development Goals.

Q) What was India's rank on the latest annual edition of the International Intellectual Property (IP) Index?

74 TELEGRAM LINK: https://t.me/opdemy WEBSITE: www.opdemy.com

A. 20 B. 30 C. 40 D. 50

Ans - C

India ranked 40 among 53 global economies on the latest annual edition of the International Intellectual Property (IP) Index released. The US Chamber of Commerce Global Innovation Policy Centre (GIPC), the Index evaluates Intellectual Property rights in 53 global economies from patent and copyright policies to commercialisation of IP assets and ratification of international treaties. The 2021 Index illustrates that economies with the most effective IP frameworks are more likely to achieve the socio-economic benefits needed to combat COVID-19, including greater access to venture capital, increased private sector investment in research and development, and over 10 times more clinical trial activity.

Q) World’s First Ship Tunnel will be built in which country?

A. China B. Norway C. Sweden D. The US

Ans - B

The Norwegian Coastal Administration has received the green signal to start working on the construction of the world’s first tunnel, being built exclusively for ships. The tunnel is dubbed as “Stad Ship Tunnel” and would be built under the mountainous Stadhavet peninsula in northwestern Norway. The 1.7 km tunnel will be able to handle ships of up to 16,000 tonnes and give ships a safer passage through the treacherous Stadhavet Sea. The tunnel will be 49 meters (161 ft) high and 36 meters (118 ft) wide. It is estimated to cost at least 2.8 billion Norwegian kroner (NZ$460 million) the construction of this ship tunnel. The construction would begin in 2022 and would take three to four years to complete.

Q) The Suez Canal has been blocked. It passes through which country?

A. Israel B. Egypt C. Lebanon D. Saudi Arabia

Ans - B

The Suez Canal, a critical shipping artery that connects the Mediterranean and Red Seas through Egypt, has been blocked after a large cargo ship ran aground while passing through it, bringing traffic on the busy trade route to a halt.

75 TELEGRAM LINK: https://t.me/opdemy WEBSITE: www.opdemy.com

A human-made waterway, the Suez Canal is one of the world’s most heavily used shipping lanes, carrying over 12% of world trade by volume. Built in 1869, it provides a major shortcut for ships moving between Europe and Asia, who before its construction had to sail around Africa to complete the same journey. The 150-year-old canal was controlled by British and French interests in its initial years but was nationalized in 1956 by Egypt’s then leader Gamal Abdel Nasser. Over the years, the canal has been widened and deepened. In 2015, Egypt announced plans to further expand the Suez Canal, aiming to reduce waiting times and double the number of ships that can use the canal daily by 2023.

Q) Chief Justice of India (CJI) SA Bobde has recommended whose name as his successor after he retires next month?

A. Justice Arun Kumar Mishra B. Justice Uday Lalit C. Justice Nuthalapati Venkata Ramana D. Justice Bhushan Ramakrishna Gavai

Ans - C

Chief Justice of India (CJI) SA Bobde has recommended Justice NV Ramana’s name as his successor after he retires next month. CJI Bobde is set to retire on 23 April. In a letter to the Law Ministry, CJI SA Bobde recommended Justice Ramana as his successor. Justice NV Ramana is the most senior judge in the apex court after CJI Bobde. On appointment, Justice NV Ramana will be the 48th Chief Justice of India and he will have a long tenure as CJI for over 16 months. The Centre had last week asked CJI Bobde to recommend his successor. Union Minister Ravi Shankar Prasad had reportedly sent a letter to Justice Bobde, seeking his recommendation, as is the practice under the Memorandum of Procedure for Appointment of Chief Justice of India. The recommendation letter has to come around a month before the vacancy arises.

Q) For the first time, the Indian Navy will be part of a French naval exercise known as 'La Perouse' in which of the following regions?

A. English Channel B. Mediterranean Sea C. Arabian Sea D. Bay of Bengal

Ans - D

For the first time, the Indian Navy will be part of a French naval exercise known as ‘La Perouse’ in the Bay of Bengal next month. The other Quad nations – the US, Japan and Australia – will also participate. This is the first time that the Indian Navy will be part of the France-led war game ‘La Perouse’. Until now, India was not invited to the French naval exercise. After the Malabar Exercise in November last year, during which the navies of India, the US and

76 TELEGRAM LINK: https://t.me/opdemy WEBSITE: www.opdemy.com

Japan got together with Australia as the new addition, India’s inclusion in the France-led joint naval drill is a significant step. “The exercise is scheduled between April 5 and 7,” said an official.

Q) With reference to the Suez Canal, consider the following statements: 1. It is one of the world’s most heavily used shipping lanes, carrying over 12% of world trade by volume. 2. Built-in 1869, it provides a major shortcut for ships moving between North America and SouthAmerica.

Which of the statements given above is/are correct?

1 only 2 only Both 1 and 2 Neither 1 nor 2

Ans - A

The Suez Canal, a critical shipping artery that connects the Mediterranean and Red Seas through Egypt, has been blocked after a large cargo ship ran aground while passing through it, bringing traffic on the busy trade route to a halt. A human-made waterway, the Suez Canal is one of the world’s most heavily used shipping lanes, carrying over 12% of world trade by volume. Built-in 1869, it provides a major shortcut for ships moving between Europe and Asia, who before its construction had to sail around Africa to complete the same journey.

77 TELEGRAM LINK: https://t.me/opdemy WEBSITE: www.opdemy.com

UPSC CSE PRELIMS 2021

CRASH COURSE DATE SUBJECT Budget and Economic 10/04/2021, 11/04/2021 Survey 12/04/2021, 13/04/2021 Art and Culture 14/0/2021, 15/04/2021, Economics 16/04/2021 17/18/2021,18/04/2021, Modern Indian History 19/04/2021 20/04/2021,21/04/2021, Geography 22/04/2021 23/04/2021,24/04/2021, Environment 25/04/2021 26/04/2021,27/04/2021, 28/04/2021,29/04/2021, Polity 30/04/2021 01/05/2021, 02/05/2021 Science and Tech *Note: Also, there will be Lectures on CSAT as and when time permits* PRICE: 1,999 For Any Queries: 7988797680, 7999136838, 6295118940

78 TELEGRAM LINK: https://t.me/opdemy WEBSITE: www.opdemy.com

Q) With reference to Electoral Bonds, consider the following statements. a) All Public sector Banks are authorized to issue and encash these bonds, which are valid for fifteen days from the date of issuance. b) The bonds are issued in multiples of Rs. 1,000, Rs. 10,000, Rs. 1 lakh, Rs. 10 lakh and Rs. 1 crore without any maximum limit.

Which of the above statements is/are correct?

A. 1 only B. 2 only C. Both 1 and 2 D. Neither 1 nor 2

Ans - B

The Supreme Court on Friday refused to stay the sale of electoral bonds prior to the Assembly elections in crucial states such as West Bengal and Tamil Nadu. A three-judge Bench, led by Chief Justice of India Sharad A. Bobde, said the scheme began in 2018 and continued in 2019 and 2020 without “any impediments”. Electoral Bond is a financial instrument for making donations to political parties. The bonds are issued in multiples of Rs. 1,000, Rs. 10,000, Rs. 1 lakh, Rs. 10 lakh, and Rs. 1 crore without any maximum limit. State Bank of India is authorized to issue and encash these bonds, which are valid for fifteen days from the date of issuance.

Q) Consider the following statements: 1. The Centre has only rejected 4.3% of all Right to Information (RTI) requests in 2019-20, the lowest ever rate, according to the Central Information Commission’s annual report. 2. The RTI Act allows public authorities to reject RTI requests on a number of grounds, ranging from information that would endanger life and safety to security and intelligence matters.

Which of the statements given above is/are correct?

A. 1 only B. 2 only C. Both 1 and 2 D. Neither 1 nor 2

Anc - C

The RTI Act allows public authorities to reject RTI requests on a number of grounds, ranging from information that would endanger life and safety to that which involves irrelevant personal information, Cabinet papers, foreign governments, copyrights, or sovereignty, security and intelligence matters. Public authorities are expected to cite the relevant clause of the Act to invoke the exemption. The Centre has only rejected 4.3% of all Right to Information (RTI) requests in 2019-20, the lowest ever rate, according to the Central Information Commission’s annual report. However,

79 TELEGRAM LINK: https://t.me/opdemy WEBSITE: www.opdemy.com

almost 40% of these rejections did not include any valid reason, as they did not invoke one of the permissible exemption clauses in the RTI Act, according to an analysis of report data by RTI activist Venkatesh Nayak. This includes 90% of rejections by the Prime Minister’s Office.

Q) Central Scrutiny Centre (CSC) and Investor Education and Protection Fund Authority (IEPFA) Mobile App is an initiative of:

A. Ministry of Corporate Affairs B. Ministry of Finance C. Ministry of Science and Technology D. None of the above

Ans - A

Union Minister for Corporate Affairs launched Central Scrutiny Centre (CSC) and Investor Education and Protection Fund Authority’s (IEPFA) Mobile App — two tech-enabled initiatives by the Ministry of Corporate Affairs (MCA).

Q) Sports Ministry has included which of the following in the Khelo India Youth Games 2021?

A. Cricket B. Kabaddi C. Yogasana D. Gili Danda

Ans - C

Sports Minister Kiren Rijiju has included Yogasana in Khelo India Youth Games 2021 in a bid to develop it as a competitive sport. Yogasana sport for both male and female categories have been included in Khelo India Youth Games, 2021. Rijiju has written a reply in the Lok Sabha that the government has also given recognition to the National Yogasana Sports Federation (NYSF) for the promotion and development of Yogasana in the country. An International Yogasana Sports Federation was formed under the presidentship of yoga guru Baba Ramdev with HR Nagendra as secretary-general in November 2019.

Q) The Bureau of Civil Aviation Security signed an LoA for civil aviation security with which international airport?

A. Indira Gandhi International Airport B. Yamuna International Airport C. Chhatrapati Shivaji International Airport D. Rajiv Gandhi International Airport

Ans - B

80 TELEGRAM LINK: https://t.me/opdemy WEBSITE: www.opdemy.com

A Letter of Agreement (LoA) was signed between the Bureau of Civil Aviation Security (BCAS) and Yamuna International Airport Private Limited (YIAPL). The LoA defines provisions for civil aviation security responsibilities and functions at the upcoming international airport at Jewar, Noida. The contract of construction of the airport has been awarded to YIAPL, which is a joint venture of the Airport Authority of India (AAI), the Government of Uttar Pradesh, and Zurich international airport.

Q) Which IIT has won the CoreNet Global Academic Challenge 6.0?

A. IIT Kharagpur B. IIT Bombay C. IIT Delhi D. IIT Roorkee

Ans - A

A team of four undergraduate students from the Indian Institute of Technology, Kharagpur emerged as winners at the CoreNet Global Academic Challenge 6.0, sponsored by Cushman and Wakefield, KI and IA: Interior Architects. The team comprised Siddhant Samarth, Pratim Majumdar, Rishita Raj and Utkarsh Agrawal and were guided by Prof. Jenia Mukherjee (Department of Humanities and Social Sciences). The team from IIT Kharagpur was one of the four teams to progress into the final round out of 1300 teams all over the world and were pitted against the University of Washington, New York University and the University of Denver in the final round hosted online on 18th March 2021. This is the first time that a squad from India has emerged as winners in the CoreNet Global Academic Challenge since the inception of the competition in 2015.

Q) Which of the following ship was commissioned into the Indian Coast Guard (ICG) by General Bipin Rawat, Chief of Defence Staff, at Chennai?

A. ICGS Vajra B. ICGS Vasuki C. ICGS Astra D. ICGS Vishal

Ans - A

L&T-built Offshore Patrol Vessel (OPV) ICGS Vajra was commissioned into the Indian Coast Guard (ICG) by General Bipin Rawat, Chief of Defence Staff, at Chennai. ICGS Vajra is the sixth vessel in the series of seven OPVs being built by L&T under a Ministry of Defence contract signed in the year 2015. Like the five OPVs in the series, ICGS Vajra was also delivered ahead of the contractual schedule. OPVs are long-range surface ships, capable of operation in maritime zones of India, including island territories with helicopter operation capabilities.

81 TELEGRAM LINK: https://t.me/opdemy WEBSITE: www.opdemy.com

Q) Who among the following virtually inaugurated the Central Scrutiny Centre (CSC) and Investor Education and Protection Fund Authority’s (IEPFA) Mobile App?

A. Kiren Rijiju B. Nirmala Sitharaman C. Piyush Goyal D. Smriti Irani

Ans - B

Union Minister for Finance & Corporate Affairs, Nirmala Sitharaman virtually inaugurated Central Scrutiny Centre (CSC) and Investor Education and Protection Fund Authority’s (IEPFA) Mobile App. The two tech-enabled initiatives have been launched by the Ministry of Corporate Affairs to leverage digital solutions to achieve the Prime Minister’s vision of ‘Digitally empowered India. The Mobile App will focus on financial literacy by spreading investors’ awareness, education, and protection among investors. The Mobile App has been developed for citizen engagement and information dissemination to promote awareness among investors.

Q) Minister of State (Independent Charge) for Civil Aviation, Hardeep Singh Puri has inaugurated the Kurnool Airport in which state?

A. Maharashtra B. Madhya Pradesh C. Telangana D. Andhra Pradesh

Ans - D

Minister of State (Independent Charge) for Civil Aviation, Hardeep Singh Puri has inaugurated the Kurnool Airport, in Andhra Pradesh virtually. The flight operations at Kurnool airport will begin on 28 March 2021. This airport has been developed under the Regional Connectivity Scheme, Ude Desh Ka Aam Nagrik (RCS-UDAN). Kurnool is the 6th airport in Andhra Pradesh to become functional after Kadapa, Visakhapatnam, Tirupati, Rajahmundry & Vijayawada.

Q) With reference to the 100+ curriculum-based comic books, consider the following statements: 1. It is an initiative of the Department of School Education and Literacy, Union Ministry of Education. 2. These comics can be accessed online on DIKSHA web portal (diksha.gov.in) or via the DIKSHA app on any android Smartphone.

Which of the statements given above is/are correct?

A. 1 only B. 2 only C. Both 1 and 2

82 TELEGRAM LINK: https://t.me/opdemy WEBSITE: www.opdemy.com

D. Neither 1 nor 2

Ans - C

Union Minister of Education launched 100+ comic books created by teachers and students of CBSE schools and curated by NCERT on 24th March 2021. Department of School Education and Literacy, in its endeavour, to provide holistic learning to students and to usher in the vision envisaged in New Education Policy 2020 has launched comic books aligned to chapters of NCERT textbooks across grades 3-12.

This innovative initiative will help in increasing the cultural and social sensitivity of our children while imparting knowledge. These comics can be accessed online on DIKSHA web portal (diksha.gov.in) or via the DIKSHA app on any android Smartphone.

Q) Consider the following statements. a) The Unique Land Parcel Identification Number (ULPIN) scheme has been launched in 10 States this year and will be rolled out across the country by March 2022 b) ULPIN database to be linked with revenue court records and is a 14-digit identification number to every plot of land in the country.

Which of the above statements is/are correct?

A. 1 only B. 2 only C. Both 1 and 2 D. Neither 1 nor 2

Ans - B

The Centre plans to issue a 14-digit identification number to every plot of land in the country within a year. It will subsequently integrate its land records database with revenue court records and bank records, as well as Aadhaar numbers on a voluntary basis, according to a parliamentary standing committee report submitted to the Lok Sabha last week. The Unique Land Parcel Identification Number (ULPIN) scheme has been launched in 10 States this year and will be rolled out across the country by March 2022, the Department of Land Resources told the Standing Committee on Rural Development.

Q) Consider the following statements: 1. As many as 27 migratory birds have been reported dead on account of avian influenza in the Pong Dam Wildlife Sanctuary 2. Pong Dam Wildlife Sanctuary is situated in Goa.

Which of the statements given above is/are correct?

A. 1 only B. 2 only

83 TELEGRAM LINK: https://t.me/opdemy WEBSITE: www.opdemy.com

C. Both 1 and 2 D. Neither 1 nor 2

Anc - A

As many as 27 migratory birds have been reported dead on account of avian influenza in the Pong Dam Wildlife Sanctuary area of Himachal Pradesh since March 25, setting off alarm bells and forcing authorities to shut down the sanctuary.

Q) With reference to the India and Bangladesh relations, consider the following statements: 1. India and Bangladesh recently signed 5 MoUs in the fields of Disaster Management, trade, NCC, ICT, and setting up of sports facilities. 2. A direct passenger train named ‘Mitali Express’ will run between Dhaka and New Jalpaiguri.

Which of the statements given above is/are correct?

A. 1 only B. 2 only C. Both 1 and 2 D. Neither 1 nor 2

Ans - C

India and Bangladesh signed 5 MoUs in the fields of Disaster Management, trade, NCC, ICT and setting up of sports facilities during the official visit of Prime Minister Narendra Modi to Bangladesh. Foundation stone was laid for infrastructure development for power evacuation facilities from the Rooppur Nuclear power plant. The Bangladesh leg of the Bangabandhu-Bapu Digital Exhibition was inaugurated by the two Prime Ministers. It will be taken to other parts of the world including the UN. Both the leaders also unveiled the foundation stone for the construction of a memorial at Ashuganj honoring the martyrs of Indian armed forces in the 1971 Liberation War of Bangladesh.

Q) India delivered on its promise of 2,00,000 doses of COVID-19 vaccines for the United Nations Peacekeeping Force (UNPKF) worldwide, sending a shipment bound for -

A. Amsterdam B. Copenhagen C. Paris D. Vienna

Ans - B

UN peacekeepers provide security and political and peacebuilding support to help countries make the difficult, early transition from conflict to peace. The United Nations Charter gives the United Nations Security Council the responsibility to

84 TELEGRAM LINK: https://t.me/opdemy WEBSITE: www.opdemy.com

maintain international peace. For this reason, the international community usually looks to the Security Council to authorize peacekeeping operations through Chapter VII authorizations. Present status: There are currently 12 UN peacekeeping operations deployed.

Q) Iran has recently signed a 25-year “strategic cooperation pact” with which country?

A. The US B. Russia C. China D. India

Ans - C

While details of the new 25-year pact were not immediately available, it includes “political, strategic and economic” components. This document can be very effective in deepening relations and would establish a blueprint for “reciprocal investments in the fields of transport, ports, energy, industry, and services”.

Q) The United Nations Human Rights Council (UNHRC) recently tabled a resolution named Resolution 46/L1 which is focused on the deteriorating human rights in which country?

A. Myanmar B. Sri Lanka C. Pakistan D. China

Ans - B

Resolution 46/L1 has decided, among other things, to “strengthen” the Office of the High Commissioner on Human Rights to collect evidence and develop possible strategies for future accountability processes for gross violations of human rights in Sri Lanka. It refers to a “persistent” lack of accountability for rights abuses committed through the years by “all parties” in Sri Lanka, including the LTTE. Most seriously, it expresses a lack of confidence in the ability of the present government in Colombo to address the shortcomings. Among the 14 countries that abstained were Japan, Indonesia, Bahrain, and Nepal. Among the 11 that voted against were China, Cuba, Pakistan, Bangladesh, Russia, and Venezuela.

Q) Which ministry has announced companies will have to disclose any holding or dealings in cryptocurrencies or virtual currencies in their statements filed with the Registrar of Companies (RoC)?

A. Ministry of Finance B. Ministry of Commerce C. Ministry of Corporate Affairs D. None of the above

Ans - C

85 TELEGRAM LINK: https://t.me/opdemy WEBSITE: www.opdemy.com

All companies will now have to disclose in their statutory financial filings to the RoC any profit or loss on transactions involving cryptocurrency, The amount of cryptocurrency held on the reporting date, and Any deposits or advances received from anyone for the purpose of investing in cryptocurrencies or virtual currencies. Some experts have said these disclosure requirements indicated the government is open to regulating cryptocurrencies instead of banning them.

Q) The Indian Cellular and Electronics Association (ICEA) has written letters to the NITI Aayog to consider declaring 2020-21 as zero years for production-linked incentive (PLI) scheme for manufacturing of -

A. Computers B. Processor C. Microchips D. Mobile Phones

Ans - D

PLI schemes for all sectors that have been announced since April last year envisage incentives on achievement of a minimum threshold of cumulative incremental investment and incremental sales of manufactured goods net of taxes. Since the government notified the PLI for mobile manufacturing on April 1 last year, FY21 was considered as the base year for the calculation of incremental investments and sales, and therefore the incentives to be given to companies.

Q) India recently signed 5 MoUs in the fields of Disaster Management, trade, NCC, ICT and setting up of sports facilities?

A. Bangladesh B. Israel C. France D. Russia

Ans - A

India and Bangladesh signed 5 MoUs in the fields of Disaster Management, trade, NCC, ICT and setting up of sports facilities during the official visit of Prime Minister Narendra Modi to Bangladesh. Foundation stone was laid for infrastructure development for power evacuation facilities from the Rooppur Nuclear power plant.

Q) With reference to Hypnea, consider the following statements: 1. They grow in the intertidal regions of the coast, namely the area that is submerged during the high tide and exposed during low tides. 2. Recently Hypnea indica and Hypnea bullata was discovered in India.

86 TELEGRAM LINK: https://t.me/opdemy WEBSITE: www.opdemy.com

Which of the statements given above is/are correct?

1 only 2 only Both 1 and 2 Neither 1 nor 2

Ans - C

Two new species of seaweed have been discovered by a group of marine biologists from Central University of Punjab, Bathinda. Named Hypnea indica (after India) and Hypnea bullata (because of the blisterlike marks on its body – bullate), the seaweeds are part of the genus Hypnea or red seaweeds. While Hypnea indica was discovered Kanyakumari in Tamil Nadu, and Somnath Pathan and Sivrajpur in Gujarat, Hypnea bullata was discovered from Kanyakumari and Diu island of Daman and Diu.

Q) Which bank has adjudged the ‘India’s Best Bank for SMEs’ at the Asiamoney Best Bank Awards 2021?

A. State Bank of India B. HDFC Bank C. ICICI Bank D. Canara Bank

Ans - B

HDFC Bank has adjudged the ‘India’s Best Bank for SMEs’ at the Asiamoney Best Bank Awards 2021. In its evaluation, the magazine said on its website – “HDFC Bank’s transformation of its micro, small and medium-sized enterprise (MSME) business over the past few years makes it a worthy winner of this award.” The Annual Best Banks survey of the financial magazine based in Hong Kong aims to identify which banks in each market have excelled across a range of core banking activities over the past 12 months. Asiamoney’s award decisions were made by a team of senior journalists after receiving detailed submissions from market participants and in conjunction with research into the banking and capital markets by our editorial committee. Senior editors also visited each country or territory to meet leading bankers and gather client and competitor feedback.

Q) The “World Development Report 2021: Data for Better Lives” was released by -

A. World Bank B. IMF C. United Nations D. Asian Development Bank Ans - A

The “World Development Report 2021: Data for Better Lives” was released by the World Bank.

87 TELEGRAM LINK: https://t.me/opdemy WEBSITE: www.opdemy.com

The World Development Report focuses on how data can improve the lives of poor people and advance development objectives, to ensure no one is left behind. It highlights the tremendous potential of the changing data landscape to improve the lives of poor people, at the same time acknowledging its potential to open back doors that can harm individuals, businesses, and societies. Through this report, World Bank aims to support its client countries by identifying where public and private sector investments are the most critical, defining a rich program for policy reform and technical assistance, and highlighting areas in which global initiatives can help to convene and facilitate cross-border cooperation.

Q) As per the Global Wind Reports 2021, which year was the best in history for the global wind industry?

A. 2018 B. 2019 C. 2020 D. 2021

Ans - C

Despite the COVID-19 pandemic, the year 2020 was the best year in history for the global wind industry as this sector installed 93GW of new capacity in 2020, according to a new report titled ‘Global Wind Report 2021, released by the Global Wind Energy Council. This is the 16th annual flagship report, released by GWEC on March 25, 2021. The 93GW represents a 53% year-on-year increase. The global wind power market increased four times in size over the past decade but the record growth was seen in 2020. This increase was driven by a surge of installations in China and the The US alone. Together, the US and China installed 75% of new installations in 2020 and account for over half of the world’s total wind power capacity.

Q) India recently concluded its two-day naval exercise PASSEX in the Bay of Bengal with which country?

A. The US B. France C. Australia D. Japan

Ans - A

India and the US kicked off a two-day naval exercise in the Bay of Bengal, reflecting the growing congruence in their defense and military partnership. The exercise, which began yesterday, will conclude today. Indian Navy deployed its warship Shivalik and long-range maritime patrol aircraft P8I in the 'PASSEX' exercise while the US Navy was represented by the USS Theodore Roosevelt carrier strike group.

88 TELEGRAM LINK: https://t.me/opdemy WEBSITE: www.opdemy.com

UPSC CSE PRELIMS 2021

CRASH COURSE DATE SUBJECT Budget and Economic 10/04/2021, 11/04/2021 Survey 12/04/2021, 13/04/2021 Art and Culture 14/0/2021, 15/04/2021, Economics 16/04/2021 17/18/2021,18/04/2021, Modern Indian History 19/04/2021 20/04/2021,21/04/2021, Geography 22/04/2021 23/04/2021,24/04/2021, Environment 25/04/2021 26/04/2021,27/04/2021, 28/04/2021,29/04/2021, Polity 30/04/2021 01/05/2021, 02/05/2021 Science and Tech *Note: Also, there will be Lectures on CSAT as and when time permits* PRICE: 1,999 For Any Queries: 7988797680, 7999136838, 6295118940

89 TELEGRAM LINK: https://t.me/opdemy WEBSITE: www.opdemy.com

Q) Who launched the ‘Tribal TB Initiative’ in pursuit of TB Mukt Bharat?

A. Narendra Modi B. Dr. Harsh Vardhan C. Arjun Munda D. Venkaiah Naidu

Ans - B

Health Minister Dr. Harsh Vardhan has launched the ‘Tribal TB Initiative’ in pursuit of TB Mukt Bharat. The government accords top priority to ensuring Universal Access to free treatment and care for TB across the country. A Guidance Note on Joint Action Plan for Tuberculosis (TB) Elimination, a Special Edition of Tribal Ministry’s Publication ‘ALEKH’ on TB, and a document on Tribal Tuberculosis (TB) Initiative was also released at the event. Over ten crore tribal population is living with a huge diversity in their population dynamics. He said the government has already increased the Budget allocation for TB in India four-fold in the last five years. Dr. Harsh Vardhan said the Union Territories of Lakshadweep and the district of Badgam in Jammu and Kashmir have been declared TB Free on World TB Day this year. During the function, Tribal Affairs Minister Arjun Munda was also present.

Q) External Affairs Minister S Jaishankar arrived in which country on Monday to attend a key conference on Afghanistan?

A. Afghanistan B. Kazakhstan C. Kyrgyzstan D. Tajikistan

Ans - D

External Affairs Minister S Jaishankar arrived in Tajikistan’s capital on Monday to attend a key conference on Afghanistan, where representatives from around 50 countries and international organizations would discuss building regional consensus around the peace process in the war-torn country. The 9th Heart of Asia-Istanbul Process (HoA-IP) ministerial conference is part of the Istanbul Process – a regional initiative on security and cooperation for a stable and peaceful Afghanistan – that was launched on November 2, 2011, in Turkey.

Q) With reference to the Indian rhinoceros, consider the following statements: 1. It is listed as critically endangered on the IUCN Red List. 2. Pobitora Wildlife Sanctuary in Assam has the highest density of Indian rhinos in the world.

Which of the statements given above is/are correct?

A. 1 only B. 2 only

90 TELEGRAM LINK: https://t.me/opdemy WEBSITE: www.opdemy.com

C. Both 1 and 2 D. Neither 1 nor 2

Ans - B

With the BJP frequently invoking Assam’s world-famous one-horned rhino to connect with the people of the state, the animal has now become a part of poll rhetoric, with promises to save “the pride of Assam”. The projection of rhino as the pride of Assam began during the Assam Movement (1979-85). Rhino started emerging as a player in the political narrative of Assam as “its poaching was linked to the control of indigenous people over land, resources, and influx of immigrants”. According to the rhino census of 2018, there are 2,413 rhinos in Kaziranga alone. It is common for every party contesting in Bokakhat and Kaliabor, the two constituencies that cover most of Kaziranga (the park spreads over six constituencies), to say that they will protect the rhino.

Q) Consider the following statements: 1. Researchers find new butterfly species of Nacaduba Sinhala Ramaswami Sadasivan, 2021 in the Agasthyamalais in the Western Ghats. 2. A person who studies or collects butterflies and moths.Lepidopterist.

Which of the statements given above is/are correct?

A. 1 only B. 2 only C. Both 1 and 2 D. Neither 1 nor 2

Ans - C

A group of lepidopterists has added a species to the expanding list of butterflies in India. The discovery of the species Nacaduba Sinhala Ramaswami Sadasivan, 2021 in the Agasthyamalais in the Western Ghats a decade ago has now found a place in the Journal of Threatened Taxa. Line Blues are small butterflies belonging to the subfamily Lycaenidae and their distribution ranges from India and Sri Lanka to the whole of southeastern Asia, Australia, and Samoa. It is the first time that a butterfly species was discovered by an all-Indian research team from the Western Ghats.

Q) Consider the following statements: 1. The Ministry of Road Transport and Highways has brought out a draft notification proposing concessions in motor vehicle tax for those who take their old vehicle for scrapping. 2. The draft proposes up to 25% concession in motor vehicle tax for non-transport vehicles and up to 15% in case of transport vehicles upon submitting a “Certificate of vehicle scrapping”.

91 TELEGRAM LINK: https://t.me/opdemy WEBSITE: www.opdemy.com

Which of the statements given above is/are correct?

A. 1 only B. 2 only C. Both 1 and 2 D. Neither 1 nor 2

Ans - C

The Ministry of Road Transport and Highways has brought out a draft notification proposing concessions in motor vehicle tax for those who take their old vehicle for scrapping. The notification dated March 29 invites objections and suggestions to the proposal for a period of 30 days. The draft proposes up to 25% concession in motor vehicle tax for non-transport vehicles and up to 15% in case of transport vehicles upon submitting a “Certificate of vehicle scrapping”. The concession will be valid for a period of 15 years for non-transport vehicles and for 8 years for transport vehicles. The concession is among several incentives planned by the Centre.

Q) The Heart of Asia – Istanbul Process (HoA-IP) is an initiative of Turkey and:

A. Afghanistan B. India C. Pakistan D. China

Ans - A

External Affairs Minister Dr. S Jaishankar reached Dushanbe on a three-day visit to Tajikistan. He will attend the 9th Ministerial Conference of Heart of Asia - Istanbul Process on Afghanistan. The Heart of Asia – Istanbul Process (HoA-IP) is an initiative of Afghanistan and Turkey, which was officially launched at a conference hosted by Turkey in Istanbul on 2 November 2011. Since then, Afghanistan supported by fourteen Participating Countries of the Heart of Asia Region and 16 Supporting Countries beyond the region as well as 12 Regional and International Organizations are leading and coordinating this Process. It is a platform for promoting regional security, economic and political cooperation centered on Afghanistan through dialogue and a set of Confidence Building Measures (CBMs).

Q) Mahinder Giri, range officer of which Tiger Reserve won the prestigious International Ranger Award for his contribution towards conservation?

A. Corbett Tiger Reserve B. Manas Tiger Reserve C. Kanha Tiger Reserve D. Rajaji Tiger Reserve

Ans - D

Union Minister for Environment has congratulated Mahinder Giri, range officer of Rajaji Tiger

92 TELEGRAM LINK: https://t.me/opdemy WEBSITE: www.opdemy.com

Reserve for being the only ranger from Asia to win the prestigious International Ranger Award for his contribution towards conservation. The award has been announced for 10 professionals across the world by the International Union for Conservation of Nature IUCN and World Commission on Protected Areas WCPA. Developed through a collaboration between the IUCN WCPA, the International Ranger Federation, Global Wildlife Conservation, and Conservation Allies, these awards, created in 2020, aim to highlight and felicitate the extraordinary work that rangers do in protected and conserved areas worldwide.

Q) The Uttar Pradesh government organised a ‘Banana Festival’ in which city?

A. Lucknow B. Kushinagar C. Kanpur D. Varanasi Ans - B

The Uttar Pradesh government organised a ‘Banana Festival’ in Kushinagar, which has witnessed the participation of at least 35 farmers and entrepreneurs. The state government had in 2018 organised the One District One Product (ODOP) scheme to promote traditional enterprise. Noting the good cultivation of bananas in Kushinagar, products in the district made from banana fibers were selected in the ODOP scheme. At least 4,000 farmers are linked to banana cultivation and after inclusion in the ODOP scheme, around 500 people are employed in its processing. There are three banana fiber processing units in the district and the construction of a Common Facility Centre (CFC) is also planned.

Q) The Health Minister of which state launched an ‘i-Learn’ Officers application for Community Health Officers?

A. Nagaland B. Mizoram C. Assam D. Tripura

Ans - A

Nagaland Health and Family Welfare Minister, S Pangnyu Phom has launched i-Learn, a capacity building and performance tracking application for Community Health Officers, CHOs in the state at his office chamber in Kohima. The state government launched the i-Learn application in partnership with USAID-NISHTHA/Jgpiego which will cater across the 189 Health & wellness centers in the state. i-Learn is a learning management system for CHOs with self-facilitated learning for improving and learning new skills in clinical and non-clinical areas. The state is committed to defeating the pandemic and as such it has rolled out several interventions including digital technology like the Naga Telehealth services. The Naga Telehealth, a telemedicine platform launched just about six months ago has received a good response from the state’s own medical doctors and CHOs wherein more than 1000

93 TELEGRAM LINK: https://t.me/opdemy WEBSITE: www.opdemy.com

teleconsultations have been conducted.

Q) Which country's parliament has passed legislation giving mothers and their partners the right to paid leave a miscarriage or stillbirth?

A. Norway B. Finland C. New Zealand D. Australia

Ans - C

New Zealand’s parliament has passed legislation giving mothers and their partners the right to paid leave following a miscarriage or stillbirth, becoming only the second country in the world to do so. India is the only other country with similar legislation.

The leave provisions apply to mothers, their partners as well as parents planning to have a child through adoption or surrogacy. One in four New Zealand women has had a miscarriage. The bill will give women and their partners time to come to terms with their loss without having to tap into sick leave. Because their grief is not a sickness, it is a loss. New Zealand was the first country in the world to give voting rights to women and has been a pioneer on issues around women's rights.

Q) Chief Justice of India (CJI) S A Bobde lauded which state’s Civil Code, and encouraged “intellectuals” indulging in “academic talk” to visit the state to learn more about it?

A. Uttar Pradesh B. Goa C. Kerala D. Madhya Pradesh

Ans - B

Chief Justice of India (CJI) S A Bobde lauded Goa’s Uniform Civil Code and encouraged “intellectuals” indulging in “academic talk” to visit the state to learn more about it. Earlier in September 2019, the Supreme Court had described Goa as a “shining example” with a Uniform Civil Code. A Uniform Civil Code is one that would provide for one law for the entire country, applicable to all religious communities in their personal matters such as marriage, divorce, inheritance, adoption etc. Article 44 of the Constitution lays down that the state shall endeavor to secure a Uniform Civil Code for the citizens throughout the territory of India.

Q) AYUSH Ministry in collaboration with which organization organised the 'Ayurveda Parav' to Promote Ayurveda as a mainstream treatment for Lifestyle-related diseases?

A. Vinoba Seva Pratishthan

94 TELEGRAM LINK: https://t.me/opdemy WEBSITE: www.opdemy.com

B. Nirmala Seva Sadan C. Shaksi World Association Charity D. Swapna Trust

Ans - A

In a bid to promote Ayurveda as the mainstream treatment for the prevalent lifestyle-related diseases, Vinoba SevaPratisthan (VSP), in collaboration with the Ministry of AYUSH, has successfully organized a 3-days “Ayurveda Paravin Bhubaneswar from 26th to 28th March 2021”.

10. With reference to the judicial review, consider the following statements: 1. Under Indian law, only a “law” can be challenged as unconstitutional. 2. Article 13(3) defines law, which includes any ordinance, order, by-law, rule, regulations, notification, custom, or usage having in the territory the force of law.

Which of the statements given above is/are correct?

1 only 2 only Both 1 and 2 Neither 1 nor 2

Ans - C

Public interest litigation has been filed in the Supreme Court by Wasim Rizvi seeking a declaration of 26 verses of the Quran as unconstitutional, non-effective, and non-functional on the ground that these promote extremism and terrorism. Under Indian law, only a “law” can be challenged as unconstitutional. Article 13(3) defines law, which includes any ordinance, order, by-law, rule, regulations, notification, custom, or usage having in the territory the force of law. “Laws in force” on the commencement of the Constitution include laws enacted by a legislature or other competent authority.

*************************************************************************************************************

95 TELEGRAM LINK: https://t.me/opdemy WEBSITE: www.opdemy.com

Be True to Yourself, Have Patience, Your efforts will bear fruit!

All The Best !

96 TELEGRAM LINK: https://t.me/opdemy WEBSITE: www.opdemy.com